You are on page 1of 95

CURRENT AFFAIRS QUESTION BANK

1. A Science and Technology Empowered Committee constituted for COVID-


19 response will be chaired by
a) Vinod Paul
b) N. K. Singh
c) Rajiv Gauba
d) Ajit Doval

Answer: A
Explanati
on
● A Science and Technology Empowered Committee for COVID-19 response
has been constituted and will be chaired by Prof. Vinod Paul, full time
member, NITI Aayog and Prof. K. Vijay Raghavan, Principal Scientific
Adviser to the government.
● It will take speedy decisions on testing facilities for COVID-19. The
committee would coordinate among science agencies, scientists, industries
and regulatory bodies.

2. CHALLENGE COVID-19 COMPETITION (C3) was launched by


a) NITI Aayog
b) Central Board of Secondary Education
c) National Council of Educational Research and Training (NCERT)
d) National Innovation Foundation

Answer: D
● The National Innovation Foundation has come up with a call inviting
innovative citizens to participate in its Challenge Covid-19 Competition
AVAIL OUR HELP ON INSTAGRAM/ TELEGRAM- DEFENCE MANIA
DOWLOAD “DEFENCE MANIA” APP
FOLLOW OUR YOUTUBE CHANNEL
CURRENT AFFAIRS QUESTION BANK

(C3).
● All interested innovators are welcome to participate with their creative
ideas and innovations for problems or issues like reducing transmission of
Coronavirus.
● National Innovation Foundation (NIF)
○ It is an autonomous body of the Department of Science and
Technology (DST), Government of India.
○ It was set up in February 2000 at Ahmedabad, Gujarat to provide
institutional support for scouting, spawning, sustaining and scaling up
the grassroots innovations across the country.

3. The term ‘Sarige Sanjeevini’, recently seen in news, is used for


a) Combination of Hydroxychloroquine and Azithromycin
b) Plant species found on Himalayas
c) Converting old buses into Sanitiser Buses
d) A species of Tortoise

ANSWER : C
Explanation:

● The Karnataka State Road Transport Corporation (KSRTC) is converting its


old buses that were slated to be scrapped into ‘sanitizer buses’ to disinfect
people in public places.
● The sanitizer buses branded as ‘Sarige Sanjeevini’ will be operated in the
city limits for the benefit of police personnel, health workers,
pourakarmikas (scavengers) and others who are carrying out essential
services during the lockdown.

AVAIL OUR HELP ON INSTAGRAM/ TELEGRAM- DEFENCE MANIA


DOWLOAD “DEFENCE MANIA” APP
FOLLOW OUR YOUTUBE CHANNEL
CURRENT AFFAIRS QUESTION BANK

4. Consider the following statements about Dragonfly Spacecraft:


a. Dragonfly is a planned spacecraft and mission of European Space Agency
that will send a mobile robotic rotorcraft lander to Titan.
b. Titan is the largest moon of Saturn and has abundant, complex, and diverse
carbon-rich chemistry on the surface.
Which of the above sentences is/are correct
(a) 1 only
(b) 2 only
(c) Both 1 and 2
(d) None of the above
ANSWER; B
EXPLANATION:

● Dragonfly is a planned spacecraft and mission of NASA, that will send a


mobile robotic rotorcraft lander to Titan, the largest moon of Saturn. It is
the fourth mission in the New Frontiers program.
● Dragonfly will be launched in 2026 and arrive in 2034 to study prebiotic
chemistry and extraterrestrial habitability at various locations where it will
perform vertical-takeoffs and landings (VTOL).
● Titan is unique in having an abundant, complex, and diverse carbon-rich
chemistry on the surface of a water-ice-dominated world with an interior
water ocean, making it a high-priority target for astrobiology and origin of
life studies.
● Titan has a nitrogen-based atmosphere like Earth. But unlike Earth, Titan
has clouds & rain of methane.

5. The recent defence agreement of Indian government for AGM-84L Harpoon


Block II air launched missiles and MK 54 All Up Round Lightweight
AVAIL OUR HELP ON INSTAGRAM/ TELEGRAM- DEFENCE MANIA
DOWLOAD “DEFENCE MANIA” APP
FOLLOW OUR YOUTUBE CHANNEL
CURRENT AFFAIRS QUESTION BANK

Torpedoes was done with which of the following country?


a) United State Of America
b) Russia
c) Israel
d) Japan

Answer: A
Explanatio
n:
● The U.S. State Department has approved two potential missile deals with
India, for an estimated
$92 million and $63 million.
● The first deal, for which Boeing is the contractor, is for 10 AGM-84L
Harpoon Block II air launched missiles, which can be fitted onto Boeing’s 8-
PI (Poseidon Eight India) maritime patrol aircraft and are intended to
enhance India’s capability in anti-surface warfare while defending its sea
lanes.
● The second deal, contracted with Raytheon Integrated Defense System for
$63 million, is for 16 MK
54 All Up Round Lightweight Torpedoes (LWT) and are expected to enhance
India’s anti- submarine warfare capability and can be used with the P-8I.
● There are no known offset agreements for both deals and any offset
agreements will be defined in negotiations between India and the
contractors.

6. Remdesivir drug, which is being studied as a Covid-19 treatment, was


originally created to fight against which of the following disease
a) Malaria
AVAIL OUR HELP ON INSTAGRAM/ TELEGRAM- DEFENCE MANIA
DOWLOAD “DEFENCE MANIA” APP
FOLLOW OUR YOUTUBE CHANNEL
CURRENT AFFAIRS QUESTION BANK

b) Ebola
c) AIDS
d) Dengue

Answer: B
Explanati
on
● Remdesivir, the drug created to fight Ebola, is being studied as a Covid-19
treatment as researchers find some promising results. It was also tried in
patients of MERS and SARS, both caused by members of the coronavirus
family, but experts said it did not show promising results back then.

● Coronaviruses have a single-strand RNA as their genetic material. When


the novel coronavirus SARS-CoV2 enters a human cell, an enzyme called
RdRP helps the virus replicate. Remdesivir works by inhibiting the activity
of RdRP.
● Gilead Science Inc’s antiviral drug remdesivir has been granted emergency
use authorization by the
U.S. Food and Drug Administration for COVID-19, clearing the way for
broader use of the drug in more hospitals around the United States.
● Data released from a trial by the National Institutes of Health (NIH) in the
United States showed that remdesivir reduced hospitalization stays by 31%
compared to a placebo treatment, but did not significantly improve
survival.
India’s Stand
● The Indian Council of Medical Research (ICMR) plans to wait and watch
for the results of WHO’s Solidarity trials to make an assessment on the
efficacy of remdesivir for COVID-19 treatment.

AVAIL OUR HELP ON INSTAGRAM/ TELEGRAM- DEFENCE MANIA


DOWLOAD “DEFENCE MANIA” APP
FOLLOW OUR YOUTUBE CHANNEL
CURRENT AFFAIRS QUESTION BANK

● However, Remdesivir is currently not available in India. The ICMR has said
it can consider using the drug if local manufacturers are willing to procure
it.
Other Lines Of Treatment That Being Investigated
● Hydroxychloroquine - an anti-malarial drug
● Ritonavir and Lopinavir - two antiviral drugs used for treatment of HIV
● Azithromycin - An Antibiotic

7. Recently, Facebook launched third party fact checking in which of the


following country
b) India
c) Pakistan
d) Sri Lanka
e) Bangladesh

Anwer: D
Explanatio
n:
● Facebook announced the launch of its third party fact checking system in
Bangladesh to discourage the spread of fake news in the country.
● Facebook, in partnership with Boom Fact check, will review and rate the
stories, including photos and videos.
● When third party fact-Checkers rate a post as false or fake, it will appear
less or at the bottom of the news feed. This will reduce the spread of the
post.
● In case a certain news item or post or video is suspected to lack
authenticity, the third party fact checkers will write articles about the

AVAIL OUR HELP ON INSTAGRAM/ TELEGRAM- DEFENCE MANIA


DOWLOAD “DEFENCE MANIA” APP
FOLLOW OUR YOUTUBE CHANNEL
CURRENT AFFAIRS QUESTION BANK

content which is shown in the related articles immediately below the story.
If a person shares a news or item with false information, they will receive a
notification about its lack of authenticity.
● Facebook started its fact-checking programme in 2016 and now it covers 60
partners in 50 languages.
● Boom is an Indian fact checking website which is available in English, Hindi
and Bengali.

8. Under Operation Sanjeevani, India has supplied essential medicines to


which of the following countries?
a) Maldives

b) Mauritius

c) Bangladesh
d) USA

Answer: A
Explanati
on
Under Operation Sanjeevani, India has supplied 6.2 tonnes of essential
medicines to Maldives.
This operation was carried out by Indian Air Force in order to provide
assistance in the fight against COVID 19.
Due to the countrywide lockdown in India, the medicines and
consumables couldn't have been transported through any other means.
Influenza vaccines and anti-viral drugs which are used in other countries to
treat COVID-19 such as Lo Indian Help to Maldives since the outbreak of COVID
In March 2020, India had also dispatched 5.5 tonnes of essential
medicines to the Maldives along with a 14-member COVID-19 Rapid
Response Team of doctors and specialists.
AVAIL OUR HELP ON INSTAGRAM/ TELEGRAM- DEFENCE MANIA
DOWLOAD “DEFENCE MANIA” APP
FOLLOW OUR YOUTUBE CHANNEL
CURRENT AFFAIRS QUESTION BANK

Indian Government has also evacuated nine Maldives nationals from


Wuhan, China, where the COVID-19 was first identified.
pinavir and Ritonavir have been delivered.
India’s Assistance to Maldives: Timeline
Operation Cactus,1988 - The Indian Forces helped the Maldives to
neutralize the coup attempt.
2004 - Several supplies were sent after the tsunami hit the Maldives.
Operation Neer, 2014 - Indian Government supplied clean drinking water to
the Maldives.

9. Who among the following has been appointed as the permanent


representative to the United Nations, recently?
a) Santosh Jha
b) TS Tirumurti
c) Deepak Mittal
d) Namrata S Kumar

Answer: B
Explanati
on
TS Tirumurti has been appointed as next permanent representative to
United Nations at New York.
Tirumurti is an Indian Foreign Service (IFS) officer of the 1985 batch and
was currently posted as secretary (economic relations).
He will replace Syed Akbaruddin, who is set to retire in April 2020.
The Covid-19 crisis and subsequent travel restrictions have delayed the
movement of several Indian envoys to postings abroad.
AVAIL OUR HELP ON INSTAGRAM/ TELEGRAM- DEFENCE MANIA
DOWLOAD “DEFENCE MANIA” APP
FOLLOW OUR YOUTUBE CHANNEL
CURRENT AFFAIRS QUESTION BANK

They are now expected to begin moving out when the nationwide
lockdown is eased next month.

10. Annual Special 301 report is published by

a) United States Trade Representative (USTR)


b) United Nations Conference on Trade and Development (UNCTAD)
c) International Labour Organization (ILO)
d) None of the above

Answer: A
Explanati
on
The United States Trade Representative (USTR) has published its Annual
Special 301 Report.
USTR identified 33 countries for Priority Watch List or Watch List in the
Special 301 Report
India Specific Observations In The Report:
India continues to be on the ‘Priority Watch List’ in the report.
Reason - For lack of adequate intellectual property (IP) rights protection
and enforcement.
It observed that India made “meaningful progress” to enhance IP
protection and enforcement in some areas over the past year.
The USTR also noted that India was ranked among the top five source
economies for fake goods
by the OECD in 2019.

However, it did not resolve recent and long-standing challenges, such as:

AVAIL OUR HELP ON INSTAGRAM/ TELEGRAM- DEFENCE MANIA


DOWLOAD “DEFENCE MANIA” APP
FOLLOW OUR YOUTUBE CHANNEL
CURRENT AFFAIRS QUESTION BANK

o innovators being able to receive, maintain & enforce patents particularly


in pharmaceutical sector;
o concerns over copyright laws not incentivising the creation and
commercialisation of content;
o an outdated trade secrets framework.

Other issues include:


o Restrictive patentability criteria to reject pharmaceutical patents
o Restrictions on transparency of information provided on
state-issued pharmaceutical
manufacturing licenses
o High customs duties on medical devices and Information and
Communications Technology
o Absence of an effective system for protecting against the unfair
commercial use
o Weak enforcement by courts and the police, and Lack of familiarity with
investigative techniques
o no centralised IP enforcement agency
The report urged India to join the Singapore Treaty on the Law of
Trademarks, a treaty that harmonises trademark registration.

11. Consider
the following statements regarding International Atomic Energy
Agency (IAEA):
a. IAEA is an autonomous intergovernmental organization dedicated to
increasing the contribution of atomic energy to the world’s peace.
b. Its Secretariat is headquartered in Vienna, Austria.
c. It is also known as the world's "Atoms for Peace"
organization. Which of the above statements is/are
Correct?
AVAIL OUR HELP ON INSTAGRAM/ TELEGRAM- DEFENCE MANIA
DOWLOAD “DEFENCE MANIA” APP
FOLLOW OUR YOUTUBE CHANNEL
CURRENT AFFAIRS QUESTION BANK
a) 1 and 2 Only
b) 2 and 3 Only
c) 1 and 3 Only
d) 1, 2 and 3

Answer: D

Explanati
on
The IAEA, U.N. atomic agency, is sending an initial batch of Coronavirus
testing gears to about 40 countries. The organization had recently received
requests from 90 countries for assistance with COVID-19 testing.
It will enable them to perform a standard test for the coronavirus
involving a technique derived from nuclear science.
Dozens of laboratories in Africa, Asia, Europe, Latin America and the
Caribbean will receive diagnostic machines and kits, reagents and
laboratory consumables to speed up national testing.

They will also receive biosafety supplies, such as personal protection


equipment (PPE) and laboratory cabinets for the safe analysis of collected
samples.
Technique Used By Kits
Kits use the technique called real time reverse transcription-polymerase
chain reaction or RT- PCR.
It detects the coronavirus's RNA, its genetic fingerprint, on a swab sample.
It is commonly used in developed countries to tell whether someone is
infected.
About International Atomic Energy Agency (IAEA)
AVAIL OUR HELP ON INSTAGRAM/ TELEGRAM- DEFENCE MANIA
DOWLOAD “DEFENCE MANIA” APP
FOLLOW OUR YOUTUBE CHANNEL
CURRENT AFFAIRS QUESTION BANK

It is an autonomous intergovernmental organization dedicated to


increasing the contribution of atomic energy to the world’s peace and
well-being.
It also ensures that agency assistance is not used for military purposes.
It was set up as the world's "Atoms for Peace" organization in 1957
within the United Nations family.
The IAEA Secretariat is headquartered at the Vienna International
Centre in Vienna, Austria. Relationship With United Nations
As an independent international organization related to the United Nations
(UN) system, the IAEA's
relationship with the UN is regulated by a special agreement.
In terms of its Statute, the IAEA reports annually to the UN General
Assembly and, when appropriate, to the UN Security Council regarding
States' non-compliance with safeguards obligations, as well as on matters
relating to international peace and security.

12. Consider the following statements regarding the World Games:


a. Organizsed by the International Olympic Committee, the World Games is a
multi-sport event that is held every 4 Years.
b. World Games 2021 was scheduled to be hosted by
Birmingham, United States. Which of the above statements is/are
Incorrect?
a) 1 Only

b) 2 Only

c) Both 1 and 2
d) Neither 1 nor 2
Answer: A

AVAIL OUR HELP ON INSTAGRAM/ TELEGRAM- DEFENCE MANIA


DOWLOAD “DEFENCE MANIA” APP
FOLLOW OUR YOUTUBE CHANNEL
CURRENT AFFAIRS QUESTION BANK

Explanati
on
The World Games 2021 has been postponed by nearly an year to avoid a
clash with Tokyo Olympic Games.
The 11th Edition of the World Games had originally been planned to take
place from 15–25 July 2021. Now it will be held from 7th to 17th July
2022.
The 11th Edition of the World Games 2021 will be hosted by
Birmingham, United States. About the World Games
Organizsed by the International World Games Association (IWGA)
A multi-sport event that is held every 4 Years, typically an year after the
Summer Olympics.
It includes sporting events for 32 sports that are not part of the Olympic
programme.
It was first held in 1981 and so far, it has not yet been held in African
countries.

13. Consider the following statements:


a. Exercise Pitch Black is a biennial multilateral air combat exercise hosted by
the Royal Australian Air Force.
b. AUSINDEX is a biennial bilateral maritime exercise between Indian Navy
and Royal Australian Navy.
Which of the above statements is/are Correct?
a) 1 Only
b) 2 Only
c) Both 1 and 2
d) Neither 1 nor 2

AVAIL OUR HELP ON INSTAGRAM/ TELEGRAM- DEFENCE MANIA


DOWLOAD “DEFENCE MANIA” APP
FOLLOW OUR YOUTUBE CHANNEL
CURRENT AFFAIRS QUESTION BANK

Answer: C
Explanati
on
Exercise Pitch Black was cancelled due to COVID-19 threats. This exercise is
a biennial multilateral air combat exercise hosted by the Royal Australian
Air Force.

Main objective of this exercise is to practice Offensive Counter Air and


Defensive Counter Air combat in a simulated war environment.
The first exercise took place in 1981. India for the first time participated in
this exercise in 2018.
In the last edition of Pitch Black 2018, the Indian Air Force (IAF) for the
first-time deployed fighter aircraft - Su-30MKI fighters, C-130 and C-17
transport aircraft.
AUSINDEX – Joint naval exercise between India and Australia

14. Noor, a military satellite, has been launched by


a) Saudi Arab
b) Iran
c) Pakistan
d) Oman

Answer: B
Explanati
on
Iran claims to have launched its first military satellite into orbit. According
to Iran, the satellite called Noor was launched aboard the Qased, Iran's

AVAIL OUR HELP ON INSTAGRAM/ TELEGRAM- DEFENCE MANIA


DOWLOAD “DEFENCE MANIA” APP
FOLLOW OUR YOUTUBE CHANNEL
CURRENT AFFAIRS QUESTION BANK

first-ever three-stage launch vehicle, and placed into orbit a 425 km above
the Earth.
It was launched from Iran’s Central Desert and reached an altitude of 425
kilometers, or 265 miles, above the Earth.
This is being termed as Iran’s first successful rocket launch in some time
after repeated failed attempts to put a vehicle into space. Iran has suffered
a string of launch failures since 2017.
Although this was not the first successful satellite launch for Iran. The
country put up its first communications satellite in 2009.
Analysis
There is technological overlap between space launches and
intercontinental ballistic missiles (ICBMs).
Hence, western powers are criticising Iran as they believe that the recent
space launch will enable Iran to strengthen its missile programme.
However, Iran denies that the aim of its space programme is to pursue
ICBM technology.
Analysts, on the other hand, believe that through its space launch, Iran is
achieving the objective of
flexing muscle without giving the US the legal grounds to go to the UNSC.

15. Consider
the following statements regarding Mars Mission of different
countries:
a. Tianwen-1 is the mars exploration mission of China.
b. So far, United States, Russia and India have been able to launch
successful Mars Mission. Which of the following statements is/are correct?
a) 1 Only

b) 2 Only

AVAIL OUR HELP ON INSTAGRAM/ TELEGRAM- DEFENCE MANIA


DOWLOAD “DEFENCE MANIA” APP
FOLLOW OUR YOUTUBE CHANNEL
CURRENT AFFAIRS QUESTION BANK

c) Both 1 and 2
d) Neither 1 nor 2

Answer: A
Explanati
on
China has named its first Mars exploration mission as ‘Tianwen-1’ (means
Heavenly Questions or Questions to Heaven).
This mission will include an orbiter, a lander and a rover and is expected
to be launched later this year.
With the mission, China aims to catch up with the other nations that have
launched successful mars missions including the United States, European
Union, Russia and India.
India and Mars
India became the first Asian country to have successfully launched its
Mars orbiter mission, Mangalyaan which has entered the orbit of the red
planet in 2014.
India also became the first country to have entered the Martian orbit in its
first attempt.

16. Which of the following countries has become first Arab country to legalise
cannabis farming?
a) Oman
b) Jordan
c) Yamen
d) Lebanon
AVAIL OUR HELP ON INSTAGRAM/ TELEGRAM- DEFENCE MANIA
DOWLOAD “DEFENCE MANIA” APP
FOLLOW OUR YOUTUBE CHANNEL
CURRENT AFFAIRS QUESTION BANK

Answer: D
Explanati
on
Lebanon has become first Arab country to legalise cannabis farming for
medical & industrial purposes.
This step has been taken in order to generate lucrative exports and
foreign currency as the country struggles to cope with a financial crisis.
Background
The idea of legalising cannabis cultivation to produce high value-added
medicinal products for export was explored in a report by McKinsey that
was commissioned by the Lebanese government in 2018.
The consultancy firm reportedly estimated that the industry could generate
as much as $1bn annually.
Current Financial Crisis In Lebanon
The country is in the grip of an unprecedented financial crisis that sparked
an uprising in October.
The local currency has nearly halved in value since then, while a crippling
dollar shortage has impacted imports. Prices have soared, and
unemployment has risen sharply.
The arrival of the coronavirus and the subsequent strict lockdown has
made matters worse.
Lebanon and Cannabis
Although growing the plant was previously illegal, cannabis has long been
farmed openly in the fertile and impoverished Bekaa Valley.

According to the United Nations, the country is the world’s third largest
supplier of cannabis resin, or hashish, after Morocco and Afghanistan.

AVAIL OUR HELP ON INSTAGRAM/ TELEGRAM- DEFENCE MANIA


DOWLOAD “DEFENCE MANIA” APP
FOLLOW OUR YOUTUBE CHANNEL
CURRENT AFFAIRS QUESTION BANK

17. Operation Meghdoot is related to


a) Bringing Artificial Rain in parts of Karnataka
b) Atmospheric study to forecast Indian Monsoon
c) Combat Operation against Naxalites in India
d) The capturing of Siachen glacier

Answer: D
Explanati
on
On April 13, the Indian Army observed the 36th Siachen Day,
remembering the gallant martyrs of Siachen.
Siachen Day commemorates the courage and fortitude displayed by
troops of the Indian Army in securing the highest and coldest battlefield
in the world.
On April 13, 1984, the Indian Army launched ''Operation Meghdoot'' to
secure Bilafond La and other passes on the Saltoro Ridgeline from
Pakistani aggression.
Background:
Siachen lies in the Karakoram Range in North-West India. Siachen Glacier
is 76.4 km long and covers about 10,000 sq km of uninhabited terrain.
It was in 1974 that Pakistan started permitting mountaineering expeditions
in the Siachen Glacier.
By the spring of 1983, it was clear that India needed to maintain a close
watch on Siachen.
The conflict began when Pakistan allowed Japanese expedition to scale
Rimo I (an important peak in the region) in 1984. This led to the launch of
Operation Meghdoot.

AVAIL OUR HELP ON INSTAGRAM/ TELEGRAM- DEFENCE MANIA


DOWLOAD “DEFENCE MANIA” APP
FOLLOW OUR YOUTUBE CHANNEL
CURRENT AFFAIRS QUESTION BANK

18. Recently, the Army Medical Corps Raising Day was celebrated. It was
celebrated under the motto
a) Aarogyam Bhavatu
b) Stay Fit, Stay Healthy
c) Survey Santu Nirmaya
d) None of the above

Answer: C
Explanati
on
On April 3, the Army Medical Corps (AMC) celebrated its 256th Raising Day.
It was raised based on the Royal Medical Corps.
It was held with the motto 'Survey Santu Nirmaya', meaning 'Let all
humans be free from disease and disability'.

About Army Medical Corps


It is a specialist corps in the Indian Army which primarily provides medical
services to all Army personnel, serving and veterans, along with their
families.
It came into existence as a homogeneous corps of officers and men on the
pattern of the Royal Army Medical Corps on 3 Apr 1943 by the
amalgamation of the Indian Medical Service, the Indian Medical
Department and the Indian Hospital & Nursing Corps.
It has excelled in all fields, be it warzone, foreign missions, peacetime
assignments or humanitarian assistance with aid to civil authorities in
emergency situations.

AVAIL OUR HELP ON INSTAGRAM/ TELEGRAM- DEFENCE MANIA


DOWLOAD “DEFENCE MANIA” APP
FOLLOW OUR YOUTUBE CHANNEL
CURRENT AFFAIRS QUESTION BANK

19. Trends in World Military Expenditure is an annual report published by


a) Stockholm International Peace Research Institute
b) United Nations Disarmament Commission
c) International Peace Institute
d) None of the above

ANSWER : A

EXPLANATION:
The Stockholm International Peace Research Institute (SIPRI) has released the
“Trends in World Military Expenditure 2019” Report.
India Specific Observation
India has emerged as the third
largest military spender in the
world, just behind the US and
China.
Expenditure in 2019 - $71.1 billion
India’s expenditure grew by 6.8
per cent from 2018 and the
country climbed up the rankings
from its earlier fourth spot.
India’s tensions and rivalry with
Pakistan and China were among
the major drivers for its increased
military spending.
Other reasons include -
increasing salary of nearly 1.4
million personnel serving in the
security forces and pensions for

AVAIL OUR HELP ON INSTAGRAM/ TELEGRAM- DEFENCE MANIA


DOWLOAD “DEFENCE MANIA” APP
FOLLOW OUR YOUTUBE CHANNEL
CURRENT AFFAIRS QUESTION BANK

more than 2 million former


military personnel
India’s military expanse during 2010-19 decade grew by 37 per cent.
However, the country’s military burden fell from 2.7 per cent of the Indian
GDP in 2010 to 2.4 per cent in 2019.
Its rival Pakistan’s military expenditure rose by 70 per cent over the decade
to reach $10.3 billion, with the burden rising from 3.4 per cent of GDP in
2010 to 4 per cent in 2019.

20. The Abel Prize 2020 has been awarded to


a) Hillel Furstenberg
b) Gregory Margulis
c) Both A and B
d) Neither A Nor B

Answer: C
Explanati
on
The Abel Prize for the year 2020 has been given to the two great
mathematicians Hillel Frustenberg and Gregory Margulis.
They have been given this award for pioneering the use of methods from
probability and dynamics in group theory, number theory and
combinatorics.

The Abel prize, named after Niels Henrik Abel, a Norwegian


mathematician, is set up more like the Nobels.
Since 2003 it has been given annually to highlight important advances in

AVAIL OUR HELP ON INSTAGRAM/ TELEGRAM- DEFENCE MANIA


DOWLOAD “DEFENCE MANIA” APP
FOLLOW OUR YOUTUBE CHANNEL
CURRENT AFFAIRS QUESTION BANK

mathematics.
The prize is awarded by the Norwegian Academy of Science and Letters,
which has appointed an Abel Committee consisting of five mathematicians
to review the nominated candidates and submit a recommendation for a
worthy Abel laureate.

21. Which of the following Indian Mountaineer has entered Limca Book of
Records?
a) Satyarup Siddhanta
b) Bachendri Pal
c) Santosh Yadav
d) Malavath Poora

Answer: A
Explanati
on
Satyarup Siddhanta has been nominated for the coveted Limca Book of
World Records for his extra- ordinary achievement to be the first Indian to
climb the highest volcanoes of each of the 7 continents.
He climbed the highest volcano of Antarctica - Mt Sidley to complete his 7
summits and Volcanic 7 summits.
He also holds a world record for being the youngest in the world to summit
the highest mountains (7 Summits) of each continent, including the Mt.
Everest in Nepal.
His Seven volcanic summits are: Ojos del Salado (6,893 m) - Chile - South
America, Mt Kilimanjaro (5,895 m)

AVAIL OUR HELP ON INSTAGRAM/ TELEGRAM- DEFENCE MANIA


DOWLOAD “DEFENCE MANIA” APP
FOLLOW OUR YOUTUBE CHANNEL
CURRENT AFFAIRS QUESTION BANK

- Tanzania - Africa, Mount Elbrus (5,642 m) - Russia - Europe, Mount Pico


de Orizaba (5,636 m) - Mexico - North America, Mt Damavand (5,610 m) -
Iran - Asia, Mt Giluwe (4,368 m) - Papua New Guinea - Australia and Mt
Sidley (4,285 m) - Antarctica.

22. Consider
the following statements:
a. MACS 4028 is a variety of rice.
b. Biofortification improves nutritional quality of crops during the
plant growth phase. Which of the statements given above is/are
correct?
a) 1 only

b) 2 only

c) Both 1 and 2
d) Neither 1 nor 2

Answer: B
Explanati
on
Scientists from an autonomous institute under the Department of Science
& Technology in Pune have developed a biofortified wheat variety MACS
4028, which shows high protein content.

The wheat variety developed shows high protein content of about 14.7%,
better nutritional quality having zinc 40.3 ppm, and iron content of
40.3ppm and 46.1ppm respectively, good milling quality and overall
acceptability.
MACS 4028
MACS 4028, is a semi-dwarf variety, which matures in 102 days and has
AVAIL OUR HELP ON INSTAGRAM/ TELEGRAM- DEFENCE MANIA
DOWLOAD “DEFENCE MANIA” APP
FOLLOW OUR YOUTUBE CHANNEL
CURRENT AFFAIRS QUESTION BANK

shown the superior and stable yielding ability of 19.3 quintals per hectare.
It is resistant to stem rust, leaf rust, foliar aphids, root aphids, and brown
wheat mite.
It is also included by the Krishi Vigyan Kendra (KVK) programme for United
Nations Children's Fund (UNICEF) to alleviate malnutrition in a sustainable
way
It has also been notified by the Central Sub-Committee on Crop Standards,
Notification and Release of Varieties for Agricultural Crops (CVRC) for
timely sown, rainfed condition of Peninsular Zone, comprising
Maharashtra and Karnataka.
Biofortification
Fortification is the practice of deliberately increasing the content of an
essential micronutrient, i.e. vitamins and minerals (including trace
elements) in a food, so as to improve the nutritional quality of the food
supply and provide a public health benefit with minimal risk to health.
Biofortification is the process by which the nutritional quality of food crops
is improved through agronomic practices, conventional plant breeding, or
modern biotechnology.
Biofortification differs from conventional fortification as it aims to increase
nutrient levels in crops during plant growth rather than through manual
means during processing of the crops.
Biofortification thus presents a way to reach populations where
supplementation and conventional fortification activities may be difficult
to implement.

23. Considerthe following statements about Sahyadri Megha:


a. Sahyadri Megha is a variety of paddy.
b. Sahyadri Megha is resistant to blast
disease. Which of the statements given
AVAIL OUR HELP ON INSTAGRAM/ TELEGRAM- DEFENCE MANIA
DOWLOAD “DEFENCE MANIA” APP
FOLLOW OUR YOUTUBE CHANNEL
CURRENT AFFAIRS QUESTION BANK

above is/are correct?


a) 1 only

b) 2 only

c) Both 1 and 2
d) Neither 1 nor 2

Answer: C
Explanati
on
As part of its initiative to prevent decline in the area under paddy
cultivation, the University of Agricultural and Horticultural Sciences
(UAHS), Shivamogga, Karnataka has developed ‘Sahyadri Megha’, a new
red variety of paddy that is resistant to blast disease and rich in nutrients.
As the Jyothi variety, widely cultivated in the command areas of the Bhadra
and the Tunga reservoirs, had become vulnerable to blast disease and other
infestations, there was a demand for a new paddy variety that is resistant
to the infestations.

Another objective of developing the new variety was to cater to the strong
demand for red rice, rich in fibre and protein, by health-savvy consumers
in urban areas.
The new variety has been approved by the State-level Seed Sub-Committee
and will be available for farmers from the coming kharif season. It will also
be notified under the Indian Seed Act 1966 shortly, after which it will
become part of the seed chain.
Sahyadri Megha
Sahyadri Megha is developed under the hybridization breeding method by
cross-breeding the best among the ‘Jyothi’ variety with that of ‘Akkalu’, a

AVAIL OUR HELP ON INSTAGRAM/ TELEGRAM- DEFENCE MANIA


DOWLOAD “DEFENCE MANIA” APP
FOLLOW OUR YOUTUBE CHANNEL
CURRENT AFFAIRS QUESTION BANK

native disease-resistant and protein-rich paddy variety.


The protein content in Sahyadri Megha is 12.48%, higher than the other red
rice varieties grown. Also the yield per hectare from it is around 65 quintals,
substantially higher than other red paddy varieties.
As the new variety can be harvested after 120 days of sowing, it is a
medium-term paddy that can be grown when there is a delay in the onset of
monsoon. Moreover, as the variety is disease-resistant, the cultivation cost
will also be low.

24. Consider the following statements regarding RaIDer-X


a. RaIDer-X can detect up to 20 explosives from a stand-off distance of two
meters.
b. It has been developed by the Ministry of Home Affairs to
combat terrorism. Which of the above statements is/are correct?
(a) 1 only

(b) 2 only

(c) Both 1 and 2


(d) Neither 1 nor 2

Answer: A
Explanatio
n:
● A new explosive detection device, RaIDer-X, was unveiled at the National
Workshop on Explosive Detection (NWED-2020) in Pune (Maharashtra).
● It has been co-developed by High Energy Materials Research Laboratory
(HEMRL), Pune and Indian Institute of Science(IISc), Bangalore.
● RaIDer-X can detect up to 20 explosives from a stand-off distance of two
meters.
AVAIL OUR HELP ON INSTAGRAM/ TELEGRAM- DEFENCE MANIA
DOWLOAD “DEFENCE MANIA” APP
FOLLOW OUR YOUTUBE CHANNEL
CURRENT AFFAIRS QUESTION BANK

● It has various applications including narcotics, for local police, for customs
and other detection agencies who need to detect various elements which
may be explosive or non-explosive in nature.
● A data library can be built in the system to expand its capability which
helps to detect a number of explosives in pure form as well as with the
contaminants.
● High Energy Materials Research Laboratory is a premier laboratory of
Defence Research and Development Organisation(DRDO).

c. With reference to Yuva Vigyani Karyakram (YuViKa), consider the following


statements: The Program is primarily aimed at imparting basic knowledge
on Space Technology, Space Science and Space Applications to the younger
ones.

25. Which of the statements given above is/are correct?


a) 1 only
b) 2 only
c) Both 1 and 2
d) Neither 1 nor 2

Answer: B
Explanatio
n:
● ISRO has shortlisted 358 high school students from across the country to be
part of its second annual
‘catch them young’ programme, YuViKa.
● YuViKa is a special programme for School Children, launched by ISRO.
● It is in tune with the Government’s vision “Jai Vigyan, Jai Anusandhan”.
● The Program is primarily aimed at imparting basic knowledge on Space
AVAIL OUR HELP ON INSTAGRAM/ TELEGRAM- DEFENCE MANIA
DOWLOAD “DEFENCE MANIA” APP
FOLLOW OUR YOUTUBE CHANNEL
CURRENT AFFAIRS QUESTION BANK

Technology, Space Science and Space Applications to the younger ones


with the intent of arousing their interest in the emerging areas of Space
activities.
● Participation:
a) 3 students from each State/ Union Territory will be selected to
participate in this program. 5 additional seats are reserved for OCI
candidates across the country.
b) The students are selected based on their performance in the 8th
Standard (covering CBSE, ICSE and State syllabus)
● Programmes like YUVIKA might produce a good number of scientists in
the coming years and would lead to scientific development and nation
building.

26. The first supercomputer assembled indigenously is known as


a. Param Shivay
b. Param Shakti
c. Param Yuva
d. Param Ishan

Answer: A
Explanati
on
The first supercomputer assembled indigenously, called Param Shivay, was
installed in IIT (BHU) and was inaugurated by the Prime Minister.
National Supercomputing Mission
● The Mission, approved in 2016, envisages empowering our national
academic and R&D institutions spread over the country by installing a
vast supercomputing grid comprising more than 70 high- performance
AVAIL OUR HELP ON INSTAGRAM/ TELEGRAM- DEFENCE MANIA
DOWLOAD “DEFENCE MANIA” APP
FOLLOW OUR YOUTUBE CHANNEL
CURRENT AFFAIRS QUESTION BANK

computing facilities.

27. YuViKa is a special programme for School Children, launched by the Ministry
of Science and Technology.

● These supercomputers will also be networked on National


Supercomputing grid over the National Knowledge Network (NKN). The
NKN is another programme of the government which connects academic
institutions and R&D labs over a high speed network.
● The Mission also includes development of highly professional High-
Performance Computing (HPC) aware human resource for meeting
challenges of development of these applications.
● he Mission would be implemented by Department of Science and
Technology (Ministry of Science and Technology) & Ministry of Electronics
and Information Technology (MeitY), through Centre for Development of
Advanced Computing (C-DAC), Pune & Indian Institute of Science (IISc),
Bengaluru.
● Currently there are four supercomputers from India in the Top 500 list of
supercomputers in the world.

28. On which of the following disease Hydroxychloroquine and Chloroquine are


prescribed?
a) Tuberculosis
b) Malaria
c) Kala Azar
d) Dengu
Answer: B
Explanatio
AVAIL OUR HELP ON INSTAGRAM/ TELEGRAM- DEFENCE MANIA
DOWLOAD “DEFENCE MANIA” APP
FOLLOW OUR YOUTUBE CHANNEL
CURRENT AFFAIRS QUESTION BANK

n:
Hydroxychloroquine and chloroquine are oral prescription drugs that have
been used for treatment of malaria and certain inflammatory conditions.
● Malaria, caused by a plasmodium parasite, is a mosquito-borne infectious
disease that affects humans and other animals.
● Chloroquine was derived from quinine, isolated from the cinchona tree
barks by French chemists in 1820 for treating fevers.
● Hydroxychloroquine is a less toxic form of chloroquine and is used for the
treatment of malaria-
specifically the type that is sensitive to chloroquine.
● It is also used for treatment of lupus, rheumatoid arthritis and porphyria
cutanea tarda.

29. Consider the following statements regarding Tejas aircraft


a. Tejas is a light combat and multi-role supersonic fighter aircraft.
b. It is manufactured by Hindustan Aeronautics Limited (HAL) in cooperation
with Russian Air Force.
c. The Tejas is the first supersonic fighter developed by Hindustan
Aeronautics Limited (HAL) Which of the above statements is/are correct?
a) 1 only

b) 1 and 2 only

c) 1 and 3 only
d) 1, 2 and 3

Answer: A
Explanatio
n:

AVAIL OUR HELP ON INSTAGRAM/ TELEGRAM- DEFENCE MANIA


DOWLOAD “DEFENCE MANIA” APP
FOLLOW OUR YOUTUBE CHANNEL
CURRENT AFFAIRS QUESTION BANK

Recently, the Defence Acquisition Council (DAC) has approved


procurement of 83 of the more advanced Mk1A version of the Tejas
aircraft. The proposal will now be placed for consideration of the Cabinet
Committee on Security (CCS.
The procurement will be a major boost to 'Make in India' as the aircraft is
indigenously designed, developed and manufactured.
The Defence Acquisition Council also accorded approval for acquisition of
indigenous Defence equipment for about Rs 1,300 crore.
Tejas
● Tejas is the Light Combat Aircraft, indigenously-designed by Aircraft
Development Agency (ADA) under the Defence Research and
Development Organisation (DRDO) and manufactured by Hindustan
Aeronautics Limited (HAL).
● It came from the Light Combat Aircraft (LCA) programme, which began in
the 1980s to replace India's ageing MiG-21 fighters. In 2003, the LCA was
officially named "Tejas".
● Tejas is a single engined, light weight, highly agile, multi-role supersonic
fighter.
● It is designed to carry a range of air-to-air, air-to-surface, precision-guided
and standoff weaponry.
● Tejas is the second supersonic fighter (Mach 1.8) developed by Hindustan
Aeronautics Limited (HAL) after the HAL HF-24 Marut.
Defence Acquisition Council
● It is the highest decision-making body in the Defence Ministry.
● It was formed, after the Group of Ministers recommendations on
'Reforming the National Security System', in 2001, post Kargil War (1999).
● The Minister of Defence is the Chairman of the Council.
● It takes decisions on new policies and capital acquisitions for the three
services (Army, Navy and Air Force) and the Indian Coast Guard.
AVAIL OUR HELP ON INSTAGRAM/ TELEGRAM- DEFENCE MANIA
DOWLOAD “DEFENCE MANIA” APP
FOLLOW OUR YOUTUBE CHANNEL
CURRENT AFFAIRS QUESTION BANK

● It accords Acceptance Of Necessity (AON) to begin acquisition proposals,


and has to grant its approval to all major deals through all their important
phases.
● It also has the power to approve any deviations in an acquisition, and
recommends all big capital defence purchases for approval of the Cabinet
Committee on Security (CCS) headed by the Prime Minister.

30. Considerthe following statements regarding Sodium Hypochlorite:


a. Sodium Hypochlorite is used as a bleaching agent.
b. Sodium Hypochlorite is used to treat
wounds like cuts. Which of the statements given
above is/are correct?
a) 1 only

b) 2 only

c) Both 1 and 2
d) Neither 1 nor 2

Answer: C
Explanati
on
Recently, in Bareily - Uttar Pradesh migrant workers travelling to their
home states, were sprayed with a disinfectant, apparently to sanitise
them.
The chemical in the spray was a sodium hypochlorite solution. This
chemical is also being used in Gujarat, Maharashtra and Punjab, for
disinfecting buildings and solid surfaces in a bid to wash away any presence
of novel coronavirus.

AVAIL OUR HELP ON INSTAGRAM/ TELEGRAM- DEFENCE MANIA


DOWLOAD “DEFENCE MANIA” APP
FOLLOW OUR YOUTUBE CHANNEL
CURRENT AFFAIRS QUESTION BANK

Sodium Hypochlorite
Sodium hypochlorite is commonly used as a bleaching agent used for a
variety of cleaning and disinfecting purposes and is also used to sanitise
swimming pools.
It releases chlorine, which is a disinfectant. The concentration of the
chemical in the solution varies according to the purpose it is meant for and
large quantities of chlorine can be harmful.
A normal household bleach usually is a 2-10% sodium hypochlorite
solution. At a much lower 0.25- 0.5%, this chemical is used to treat skin
wounds like cuts or scrapes. An even weaker solution (0.05%) is sometimes
used as a handwash.
In Delhi, officials have said a 1% sodium hypochlorite solution was used in
the spray applied on migrant workers’ belongings. The concentration in
other places, including those used on buildings or vehicles, is not very clear.
Sodium hypochlorite is corrosive, and is meant largely to clean hard
surfaces. It is not recommended to be used on human beings, certainly not
as a spray or shower. A 1% solution can cause damage to the skin of anyone
who comes in contact with it. If it gets inside the body, it can cause serious
harm to lungs. Even a 0.05% solution could be very harmful for the eyes.
Hence, in swimming pools, the quantity of sodium hypochlorite is very low,
so that it does not harm the skin.
According to public health experts, the chemical being sprayed on buildings
could be harmful to people living inside the building and they have
appealed to civic authorities to put an end to this fumigation.

Sodium Hypochlorite’s Impact On Coronavirus


The World Health Organization, and the US Centers for Disease Control and
Prevention, recommend homemade bleach solutions of about 2-10%
concentration to clean hard surfaces to clear them of any presence of the
AVAIL OUR HELP ON INSTAGRAM/ TELEGRAM- DEFENCE MANIA
DOWLOAD “DEFENCE MANIA” APP
FOLLOW OUR YOUTUBE CHANNEL
CURRENT AFFAIRS QUESTION BANK

novel coronavirus.
A Michigan State University tutorial says that cleaning hard surfaces with
this solution can disinfect them not just from novel coronavirus but also
help prevent flu, food borne illnesses, and more. However, the bleach
should be used in a well-ventilated area and gloves should be worn when
handling the product or solution.

31.Consider the following statements regarding various


constituents of blood.
a. Plasma is the largest component of blood and consists
mainly of water.
b. Red blood cells are responsible for carrying oxygen.
c. Platelets are responsible for blood
clotting. Which of the statements given
above is/are correct?
a) 1 and 2 only

b) 2 and 3 only

c) 1 and 3 only
d) 1, 2 and 3

Answer: D
Explanati
on
Recently in America a hospital took blood from a patient who had
recovered from COVID-19 two weeks ago, and infused it into another
critically ill patient. The hospital’s blood bank now plans to collect blood
from at least 250 recovered COVID-19 patients.
With no specific treatment available for novel coronavirus disease and a
AVAIL OUR HELP ON INSTAGRAM/ TELEGRAM- DEFENCE MANIA
DOWLOAD “DEFENCE MANIA” APP
FOLLOW OUR YOUTUBE CHANNEL
CURRENT AFFAIRS QUESTION BANK

vaccine at least a year away, the US Food and Drug Administration (FDA)
has approved use of blood plasma from recovered patients to treat
severely critical COVID-19 patients.
Convalescent Plasma Therapy
The said mode of treatment is called the convalescent plasma therapy. It
seeks to make use of the antibodies developed in the recovered patient
against the coronavirus.
The whole blood or plasma from such people is taken, and the plasma is
then injected in critically ill patients so that the antibodies are transferred
and boost their fight against the virus.
According to a study in The Lancet Infectious Diseases a COVID-19 patient
usually develops primary immunity against the virus in 10-14 days.
Therefore, if the plasma is injected at an early stage, it can possibly help
fight the virus and prevent severe illness, however, it is not yet clear till
what extent will it be successful.
Past Incidents Of Convalescent Plasma Therapy
The United States used plasma of recovered patients to treat patients of
Spanish flu (1918-1920) and Hong Kong used it to treat SARS patients in
2005.
In 2009, H1N1 patients were also treated with plasma and a study in
Oxford University found that convalescent plasma reduced respiratory
tract viral load and mortality in H1N1 patients.
In 2014, the World Health Organization released guidelines to treat Ebola
patients with convalescent whole blood and plasma and Democratic
Republic of Congo and Guinea eventually did so. More recently, plasma
was also used for treating MERS patients as well.
The Process
The process to infuse plasma in a patient can be completed quickly. It only
requires standard blood collection practices, and extraction of plasma.
AVAIL OUR HELP ON INSTAGRAM/ TELEGRAM- DEFENCE MANIA
DOWLOAD “DEFENCE MANIA” APP
FOLLOW OUR YOUTUBE CHANNEL
CURRENT AFFAIRS QUESTION BANK

If whole blood is donated (350-450 ml), a blood fractionation process is


used to separate the plasma. Otherwise, a special machine called
aphaeresis machine can be used to extract the plasma directly from the
donor.
While blood is extracted from the donor, the aphaeresis machine separates
and extracts the plasma using a plasma kit, and the remaining blood
components are returned into the donor’s body.
WHO guidelines in 2014 mandate a donor’s permission before extracting
plasma. Further, plasma from only recovered patients must be taken, and
donation must be done from people not infected with HIV, hepatitis,
syphilis, or any infectious disease.
If plasma needs to be collected again from the same person, it must be
done after 12 weeks of the first
donation for males and 16 weeks for females.

Status In India
India has facilities for removing 500 ml of plasma from a donor using
aphaeresis.

For this experimental therapy to be tried out, the Drug Controller General of
India will first have to grant blood banks approval for removal of plasma from
recovered COVID-19 patients.

Constituents of Blood
Blood, by definition, is a fluid that moves through the vessels of a
circulatory system. In humans, it includes plasma (the liquid portion), blood
cells (which come in both red and white varieties), and cell fragments called
platelets.
AVAIL OUR HELP ON INSTAGRAM/ TELEGRAM- DEFENCE MANIA
DOWLOAD “DEFENCE MANIA” APP
FOLLOW OUR YOUTUBE CHANNEL
CURRENT AFFAIRS QUESTION BANK

Plasma is the largest component of blood (around 55%) and consists mostly
of water (over 90%), with proteins, ions, nutrients, and wastes mixed in.
Red blood cells are responsible for carrying oxygen and carbon dioxide.
Platelets are responsible for blood clotting.
White blood cells are part of the immune system and function in immune
response.

32. Considerthe following statements:


a. Anosmia is the loss of smell.
b. Ageusia is the loss of hearing.
Which of the statements given above is/are correct?
a) 1 only
b) 2 only
c) Both 1 and 2
d) Neither 1 nor 2

Answer: A
Explanati
on
Anosmia, the loss of sense of smell, and ageusia, an accompanying
diminished sense of taste, have emerged as peculiar telltale signs of COVID-
19 and possible markers of infection.
British physicians cited reports from other countries indicating that
significant numbers of coronavirus patients experienced anosmia, saying
that in South Korea, where testing has been widespread, 30% of 2,000
patients who tested positive experienced anosmia as their major
presenting symptom (these were mild cases).
In the areas of Italy most heavily affected by the virus, doctors say they
AVAIL OUR HELP ON INSTAGRAM/ TELEGRAM- DEFENCE MANIA
DOWLOAD “DEFENCE MANIA” APP
FOLLOW OUR YOUTUBE CHANNEL
CURRENT AFFAIRS QUESTION BANK

have concluded that loss of taste and smell is an indication that a person
who otherwise seems healthy is in fact carrying the virus and may be
spreading it to others.
Anosmia
Also known as smell blindness, anosmia is the partial or complete loss of
the sense of smell. This loss may be temporary or permanent. Common
conditions that irritate the nose’s lining, such as allergies or a cold, can lead
to temporary anosmia.
More serious conditions that affect the brain or nerves or head trauma,
can cause permanent loss of smell. Old age also causes anosmia
sometimes.

Anosmia is frequently caused by a swelling or blockage in the nose that


prevents odours from getting to the top of the nose.

Ageusia
Ageusia is the loss of taste functions of the tongue. True ageusia is
relatively rare compared to hypogeusia – a partial loss of taste – and
dysgeusia – a distortion or alteration of taste.

The main causes of taste disorders are head trauma, infections of upper
respiratory tract and exposure to toxic substances and medicines.

33. Consider the following statements about Polymerase Chain Reaction (PCR)
test:
a. PCR test is used in H1N1 virus detection.
b. The PCR test uses a technique that creates copies of a segment of DNA.
c. The invention of PCR technique was awarded the Nobel
Prize for Medicine. Which of the statements given above
AVAIL OUR HELP ON INSTAGRAM/ TELEGRAM- DEFENCE MANIA
DOWLOAD “DEFENCE MANIA” APP
FOLLOW OUR YOUTUBE CHANNEL
CURRENT AFFAIRS QUESTION BANK

is/are correct?
a) 1 and 2 only

b) 2 and 3 only

c) 1 and 3 only
d) 1, 2 and 3 only

Answer: A
Explanation
:
Viral infections are mainly identified by two kinds of tests– genetic and
serological. Genetic tests can identify infections that are active, but cannot be
used to detect past infections.

RT-PCR tests (Genetic tests)


As per ICMR, designated labs are using the conventional real-time
polymerase chain reaction (PCR) test, which is conducted on swab collected
from the back of the throat, a liquid sample from the lower respiratory
tract, or a simple saliva sample.
Kary Mullis, the American biochemist who invented the PCR technique, was
awarded the Nobel Prize for Chemistry in 1993. Such tests are commonly
used in Influenza A, Influenza B and H1N1 virus detection.
The PCR test uses a technique that creates copies of a segment of DNA.
‘Polymerase’ refers to the enzymes that make the copies of DNA. The
‘chain reaction’ is how the DNA fragments are copied, exponentially — one
is copied into two, the two are copied into four, and so on.
However, SARS-COV-2 is a virus made of RNA, which needs to be converted
into DNA. For this, the technique includes a process called reverse
transcription. A ‘reverse transcriptase’ enzyme converts the RNA into DNA
AVAIL OUR HELP ON INSTAGRAM/ TELEGRAM- DEFENCE MANIA
DOWLOAD “DEFENCE MANIA” APP
FOLLOW OUR YOUTUBE CHANNEL
CURRENT AFFAIRS QUESTION BANK

and copies of the DNA are then made and amplified.


The PCR mix contains ‘reagents’ such as ‘primers’ and ‘probes’. Primers are
particular strands of DNA that are designed to bind with the DNA that is to
be copied.

While probes are used to detect the specific sequence in the DNA sample.
The fluorescent DNA binding dye called the probe shows the presence of
the virus and the test also distinguishes SARS- COV-2 from other viruses.

Antibody tests (Serological tests)

• The AFT’s principal bench is the only court that has been conducting
physical hearings since June 8 amid the Covid-19 pandemic.
• As a result, hardships were faced by armed forces personnel, retired as
well as serving, primarily due to their far-flung locations and various
security related issues.

About AFT
• It is a military tribunal of India established in 2009 under the Armed Forces
Tribunal Act, 2007.
• 169th Law Commission Report and various Supreme Court directives
formed the basisi of enactment of this act.
• Benches - The Principal Bench is located at New Delhi. There are ten
regional benches.
• Composition - The tribunal is composed of a Chairperson and two types of
members viz. Judicial and Administrative. The number of both types of
members is decided by Central Government. Normally, each bench as one
AVAIL OUR HELP ON INSTAGRAM/ TELEGRAM- DEFENCE MANIA
DOWLOAD “DEFENCE MANIA” APP
FOLLOW OUR YOUTUBE CHANNEL
CURRENT AFFAIRS QUESTION BANK

judicial member and one administrative member. Tenure of Chairpersons


and members is four years.
• Jurisdiction - The territorial jurisdiction of AFT covers entire country and
covers the three-armed forces of India viz. Army, Navy and Air Force.
Paramilitary forces don’t come under its jurisdiction.

34. Consider the following statements about BeiDou:


i. It is a Navigation Satellite System developed by China.
ii. China plans to complete it by 2024 and it can be a potential
rival to the US Global Positioning System (GPS).
iii. It consists of a total 27 satellites, all inclined geosynchronous
orbits.
iv. After completion, it will be more
accurate than GPS. Which of the above
statements is/are correct?
a) 1 only

b) 1 and 2 only

c) 1, 2, 3 and 4
d) 1, 2 and 3 only

Answer:
A
Explanati
on:
● Initiated in 1994, China has recently completed its BeiDou Navigation
Satellite System constellation, that can potentially rival the US Global
Positioning System (GPS).
● The system was named “Beidou” after the Chinese term for the Big Dipper
AVAIL OUR HELP ON INSTAGRAM/ TELEGRAM- DEFENCE MANIA
DOWLOAD “DEFENCE MANIA” APP
FOLLOW OUR YOUTUBE CHANNEL
CURRENT AFFAIRS QUESTION BANK

constellation.
● BeiDou offers services including accurate positioning, navigation and
timing as well as short message communication.
● China’s navigation system uses a network of satellites and can provide
positional accuracies of under ten metres (GPS provides positioning
accuracies of under 2.2 metres, GPS is the most accurate navigation
system in the world.).
● It consists of a total 35 satellites, 27 satellites in medium Earth orbit, five in
geostationary orbit
and three more in inclined geosynchronous orbits.

● More than 100 countries including Pakistan and countries part of the
Belt and Road Initiative (BRI) are already using the system.
● Other countries working on building their navigation systems:
○ The GPS is owned by the US government and operated by the US Air
Force. The US has provided GPS signals to users worldwide free of cost
since the 1980s.
○ Russia has its navigation system called GLONASS.
○ European Union (EU) has developed Galileo.

AVAIL OUR HELP ON INSTAGRAM/ TELEGRAM- DEFENCE MANIA


DOWLOAD “DEFENCE MANIA” APP
FOLLOW OUR YOUTUBE CHANNEL
CURRENT AFFAIRS QUESTION BANK

○ India’s navigation system is called Navigation with Indian Constellation


NAVigation with Indian Constellation (NavIC)
● The Indian Regional Navigation Satellite System (IRNSS) is officially called NavIC. The regional
geo-positioning system has been designed in India by ISRO to provide accurate positioning in
India and around the Indian mainland.
● It is designed to provide accurate position information service to users in India as well as the
region extending up to 1500 km from its boundary, which is its primary service area.
● IRNSS will provide two types of services, namely,
a) Standard Positioning Service (SPS) which is provided to all the users.
b) Restricted Service (RS) which is an encrypted service provided only to the authorised
users.
● The IRNSS System is expected to provide a position accuracy of better than 20 m in the primary
service area.
● Some applications of IRNSS are: terrestrial, aerial and marine navigation, disaster management,
vehicle tracking and fleet management and integration with mobile phones.

(NavIC).

35. What is the name given to the Oxford-AstraZeneca Covid-19 vaccine


candidate
a) Covaxin
b) Covishield
c) Coronil
d) CorOx

Answer:
B
Explanati
on
● The Drugs Controller General of India (DCGI) has approved the
application of Serum Institute of India (SII) to conduct late-stage human

AVAIL OUR HELP ON INSTAGRAM/ TELEGRAM- DEFENCE MANIA


DOWLOAD “DEFENCE MANIA” APP
FOLLOW OUR YOUTUBE CHANNEL
CURRENT AFFAIRS QUESTION BANK

trials in the country for the Covid-19 vaccine - Covishield.

● Covishield – the name given to the Oxford-AstraZeneca Covid-19 vaccine


candidate and is technically referred to as AZD1222 or ChAdOx 1 nCoV-
19.
● SII, the world’s largest maker of vaccines, has a tie-up with AstraZeneca,
the Swedish-British pharma giant, to manufacture the Covid-19 vaccine
for low- and middle-income countries.
● The Institute can now start larger phase II/III trials of the candidate in
India, ahead of Bharat Biotech’s Covaxin and Zydus Cadila’s ZyCov-D,
other candidates that are still in phase I/II trials.

36. Consider the following statements about Starship.

i. Starship prototype of the Space X’s uncrewed “Moon ship” successfully


flew to an altitude of over 500 feet for a little less than 60 seconds.
ii. It has been described as “the world’s most powerful launch vehicle”
with an ability to carry over 100 metric tonnes to the Earth’s orbit.
Which of the above statements is/are correct
a) 1 only
b) 2 only
c) Both 1 and 2
d) None of the above

Answer: B
Explanati
on:
AVAIL OUR HELP ON INSTAGRAM/ TELEGRAM- DEFENCE MANIA
DOWLOAD “DEFENCE MANIA” APP
FOLLOW OUR YOUTUBE CHANNEL
CURRENT AFFAIRS QUESTION BANK

● SN5 Starship prototype of the Space X’s uncrewed “Mars ship” successfully
flew to an altitude of over 500 feet for a little less than 60 seconds. It is a
stainless steel test vehicle and is a part of the Starship spacecraft.
● Starship, designed by SpaceX, is a spacecraft and super-heavy booster
rocket meant to act as a reusable transportation system for crew and
cargo to the Earth’s orbit, Moon and Mars.
● SpaceX has described Starship as “the world’s most powerful launch
vehicle” with an ability to carry over 100 metric tonnes to the Earth’s orbit.

● Starship has been under development since 2012 and is a part of Space X’s
central mission to make
interplanetary travel accessible and affordable and to become the first
private company to do so.
● Once functional, the Starship spacecraft will enter Mars’ atmosphere at a
speed of 7.5 km per second and will be designed to withstand multiple
entries.

● Starship can deliver satellites further and at lower marginal costs than
SpaceX’s Falcon vehicles and it can ferry both cargo and crew to the
International Space Station (ISS).
Once developed, Starship is also expected to help carry large amounts of cargo
to the Moon, for human spaceflight development and research. Beyond the
Moon, the spacecraft is being designed for carrying crew and cargo for
interplanetary missions as well

37. The two countries that has been declared measles free by WHO are
a) India and Sri Lanka
b) Sri Lanka and Maldives
c) Maldives and Bangladesh
AVAIL OUR HELP ON INSTAGRAM/ TELEGRAM- DEFENCE MANIA
DOWLOAD “DEFENCE MANIA” APP
FOLLOW OUR YOUTUBE CHANNEL
CURRENT AFFAIRS QUESTION BANK

d) Bhutan and India


Answer:
B
Explanati
on
According to WHO, measles and rubella have been eradicated from Sri Lanka
and the Maldives.
The Maldives recorded its last endemic case of measles in 2009 and rubella
in October 2015. On the other hand, Sri Lanka's last endemic cases of
measles and rubella were reported in May 2016 and March 2017
respectively.
Bhutan, North Korea and East Timor were also declared to
be measles-free. Disease Free Status
A country gets a disease-free status when there is no evidence of endemic
transmission of the viruses for over three years in the presence of a well
performing surveillance system.

38. Consider the following sentences


i. Ammonium Nitrate is a highly explosive chemical.
ii. It is used in making fertilizer and many other products also,
apart from explosive production.
iii. In India, there is no specific law/rules regarding Ammonium
Nitrate and it is regulated like other explosives under
Explosives Act, 1884.

Which of the above sentences is/are correct


AVAIL OUR HELP ON INSTAGRAM/ TELEGRAM- DEFENCE MANIA
DOWLOAD “DEFENCE MANIA” APP
FOLLOW OUR YOUTUBE CHANNEL
CURRENT AFFAIRS QUESTION BANK

a) 1 and 2 only
b) 1 and 3 only
c) 2 only
d) 1, 2, and 3

Answer: C
Explanati
on:
● A huge blast in port warehouses near central Beirut, Lebanon storing
highly explosive material killed 78 people and injured nearly 4,000 and the
likely cause appears to have been the highly reactive chemical ammonium
nitrate.
● Experts point out serious lack of maintaining safety norms at the explosive
storage. Large quantities of stored ammonium nitrate are regarded as a
major fire hazard, with multiple reported cases across the world. Over
2,750 tonnes of ammonium nitrate were kept in a warehouse, seized six
years ago.
● The smoke that came from the blast is of blood red colour. That’s because
of the nitrogen oxide air pollutants in it. While the chemicals in the air
should dissipate fairly quickly, lingering pollutants can cause problems
later, for example if they acidify rain.
Ammonium Nitrate
● Ammonium nitrate is a chemical compound with the chemical formula
NH4NO3. It is a white crystalline solid and is highly soluble in water. It is
predominantly used in agriculture as a high- nitrogen fertilizer.
● Its other major use is as a
component of explosive
mixtures used in mining,
AVAIL OUR HELP ON INSTAGRAM/ TELEGRAM- DEFENCE MANIA
DOWLOAD “DEFENCE MANIA” APP
FOLLOW OUR YOUTUBE CHANNEL
CURRENT AFFAIRS QUESTION BANK

quarrying, and civil


construction. It is also used in
production of anaesthetic gases,
cold packs and other products.
● It is the major constituent of
ANFO(ammonium nitrate/fuel
oil)), a popular industrial
explosive which account
for 80% of explosives used in North America; similar formulations have
been used in improvised explosive devices.
● Many countries are phasing out its use in consumer applications due to
concerns over its potential for misuse.
● It is not explosive on its own, rather it is an oxidiser, drawing oxygen to a
fire – and therefore making it much more intense.
● While ammonium nitrate can in fact put out a fire, if the chemical itself is
contaminated, for example with oil, it becomes highly explosive.
● However, for combinations to explode, triggers like detonators are
required. Many Improvised Explosive Devices (IEDs) used by terrorists
around the world have ANFO as the main explosive, triggered by primary
explosives like RDX or TNT.

Occurrence
● Ammonium nitrate is found as a natural mineral (gwihabaite – the
ammonium analogue of saltpetre, which is correctly called niter, and other
nitre minerals such as sodium nitrate known as nitratine) in the driest
regions of the Atacama Desert in Chile, often as a crust on the ground
and/or in conjunction with other nitrate, iodate, and halide minerals.

● Ammonium nitrate was mined there in the past, but virtually 100% of the

AVAIL OUR HELP ON INSTAGRAM/ TELEGRAM- DEFENCE MANIA


DOWLOAD “DEFENCE MANIA” APP
FOLLOW OUR YOUTUBE CHANNEL
CURRENT AFFAIRS QUESTION BANK

chemical now used is synthetic.


Regulations In India About Ammonium Nitrate
● In India, Ammonium Nitrate Rules, 2012, under Explosives Act, 1884,
define ammonium nitrate as the “compound with formula NH4NO3
including any mixture or compound having more than 45% ammonium
nitrate by weight.
● It includes emulsions, suspensions, melts or gels but exclude emulsion or
slurry explosives and non-explosives emulsion matrix and fertilizers from
which the ammonium nitrate cannot be separated.
● As it has a strong possibility of misuse, ammonium nitrate is highly
regulated in India, at least in letter.
o The manufacture, conversion, bagging, import, export, transport,
possession for sale or use of ammonium nitrate is covered under The
Ammonium Nitrate Rules, 2012. The rules also make storage of
ammonium nitrate in large quantities in populated areas illegal in India.
o For the manufacture of ammonium nitrate, an Industrial licence is
required under the Industrial Development and Regulation Act, 1951.
A license under the Ammonium Nitrate Rules, 2012 is also required for
any activity related to ammonium nitrate.

39. Consider the following sentences about Black Boxes in aircraft


i. Most aircraft are required to be equipped with two black
boxes — the cockpit voice recorder (CVR) and the flight data
recorder (FDR).
ii. The FDR records radio transmissions and other sounds in the
cockpit such as conversations between the pilots and engine
noises.
iii. The CVR records more than 80 different types of information
such as altitude, airspeed, flight heading, vertical acceleration,

AVAIL OUR HELP ON INSTAGRAM/ TELEGRAM- DEFENCE MANIA


DOWLOAD “DEFENCE MANIA” APP
FOLLOW OUR YOUTUBE CHANNEL
CURRENT AFFAIRS QUESTION BANK

pitch, roll, autopilot status etc.


iv. They are actually two blue metallic boxes containing the
recorders.
v. They are equipped towards the tail end of the aircraft – where
usually the impact of a crash is the least.

Which of the above sentences is/are correct.


a) 1, 2, 3 and 4 only
b) 1 and 5 only
c) 1, 4 and 5 only
d) 1, 2, 3, 4 and 5

Answer: B
Explanati
on:
● Most aircraft are required to be equipped with two black boxes — the
cockpit voice recorder (CVR) and the flight data recorder (FDR) — that
record the information about a flight and help reconstruct the events
leading to an aircraft accident.
● The CVR records radio transmissions and other sounds in the cockpit such
as conversations between the pilots and engine noises.
● The flight data recorder records more than 80 different types of
information such as altitude, airspeed, flight heading, vertical
acceleration, pitch, roll, autopilot status etc.
● The black boxes, which are actually two orange metallic boxes containing
the recorders, date back to the early 1950s, when, following plane crashes,
investigators were unable to arrive a conclusive cause for the accidents
and deemed it necessary to install the said recorders on aircraft. In the
AVAIL OUR HELP ON INSTAGRAM/ TELEGRAM- DEFENCE MANIA
DOWLOAD “DEFENCE MANIA” APP
FOLLOW OUR YOUTUBE CHANNEL
CURRENT AFFAIRS QUESTION BANK

initial days of the black box, the information was recorded on to a metal
strip, which was then upgraded to magnetic drives succeeded by solid
state memory chips.
● It usually takes at least 10-15 days to analyse the data recovered from the
black boxes.
How Do The Black Boxes Survive The Crash?
● The recording devices are stored inside a unit that is generally made out
of strong substances such as steel or titanium and are also insulated from
factors such as extreme heat, cold or wetness.
● To protect these black boxes, they are equipped towards the tail end of
the aircraft – where usually the impact of a crash is the least.
● There have been cases where planes have crashed into water bodies. To
make black boxes discoverable in situations when they are under water,
they are equipped with a beacon that sends out ultrasound signals for 30
days. However, in certain cases – like the Malaysian Airlines MH370 flight
– the recorders aren’t found despite all the redundancies.

In September 2019, member countries of WHO South-East Asia Region had


set 2023 as target for elimination of measles and rubella.
Status in India
India has a long road ahead, particularly because vaccine-resistant voices are
sometimes being heard.
For the period between May 2018 and April 2019, India reported 47,056
measles cases and 1,263 rubella cases during these 12 months. India
currently gives a measles rubella vaccine in its universal immunisation
programme to tackle both measles and rubella.
It initiated the world’s largest Measles-Rubella (MR) Campaign, in
February 2017, targeting vaccination of 410 million children and
adolescents aged between 9 months and 15 years.

AVAIL OUR HELP ON INSTAGRAM/ TELEGRAM- DEFENCE MANIA


DOWLOAD “DEFENCE MANIA” APP
FOLLOW OUR YOUTUBE CHANNEL
CURRENT AFFAIRS QUESTION BANK

Measles
● Measles is a highly contagious, serious disease caused by a virus in the
paramyxovirus family and it is normally passed through direct contact and
through the air. The virus infects the respiratory tract, then spreads
throughout the body. Measles is a human disease and is not known to occur
in animals.
● Even though a safe and cost-effective vaccine is available, in 2018, there
were more than 140 000 measles deaths globally, mostly among children
under the age of five.

Rubella
● Rubella is an acute, contagious viral infection. While rubella virus infection
usually causes a mild fever and rash in children and adults, infection during
pregnancy, especially during the first trimester, can result in miscarriage,
fetal death, stillbirth, or infants with congenital malformations, known as
congenital rubella syndrome (CRS).
● The rubella virus is transmitted by airborne droplets when infected people
sneeze or cough. Humans are the only known host.
The Measles & Rubella Initiative
● Launched in 2001, the Measles & Rubella Initiative (M&R Initiative) is a
global partnership led by the American Red Cross, United Nations
Foundation, Centres for Disease Control and Prevention (CDC), UNICEF
and WHO.
● The Initiative is committed to ensuring that no child dies from measles or
is born with congenital rubella syndrome. The Initiative helps countries to
plan, fund and measure efforts to stop measles and rubella for good.

40. Recently, work begun on world's longest subsea power cable. This power
cable runs between
AVAIL OUR HELP ON INSTAGRAM/ TELEGRAM- DEFENCE MANIA
DOWLOAD “DEFENCE MANIA” APP
FOLLOW OUR YOUTUBE CHANNEL
CURRENT AFFAIRS QUESTION BANK

a) Germany & Denmark


b) France & Germany
c) France & UK
d) UK & Denmark

Answer –
D
Explanati
on:
Construction work has begun in Lincolnshire on the world’s longest subsea
power cable.
This power cable will run between Britain and Denmark to share
renewable energy between the two countries.
The 475-mile (765km) cable is a joint-venture between National Grid in
the UK and Denmark’s Energinet.
The UK has about 5GW of power cable capacity connecting Britain’s
electricity system to power generated in the Netherlands, France and the
Republic of Ireland.

41. The first-ever military communications satellite ANASIS-II has been launched
by
a) North Korea
b) South Korea
c) Iran
d) Turkmenistan

Answer –
B
AVAIL OUR HELP ON INSTAGRAM/ TELEGRAM- DEFENCE MANIA
DOWLOAD “DEFENCE MANIA” APP
FOLLOW OUR YOUTUBE CHANNEL
CURRENT AFFAIRS QUESTION BANK

Explanati
on
South Korea launched its first-ever military communications satellite. The
military satellite, ANASIS- II is aimed at enhancing the nation’s ability to
defend itself against nuclear-armed North Korea.
The launch of ANASIS-II satellite has made South Korea the 10th country
in the world to own a military-only communications satellite.
The satellite was launched by private operator SpaceX.

42. Consider the following statements regarding Malabar Exercise:


i. Malabar is a trilateral military exercise between India, USA and
Japan.
ii. The annual Malabar series began in 1992 as a bilateral
exercise between India and the United States.
Which of the above statements is/are Correct?
a) 1 Only
b) 2 Only
c) Both 1 and 2
d) Neither 1 nor 2

Answer –
C
Explanati
on
India is planning to invite Australia to join its high-level naval exercise named
Malabar Exercise. If a green signal is given then it will be the first time since
2007 that all members of Quad will participate in a joint military drill.
AVAIL OUR HELP ON INSTAGRAM/ TELEGRAM- DEFENCE MANIA
DOWLOAD “DEFENCE MANIA” APP
FOLLOW OUR YOUTUBE CHANNEL
CURRENT AFFAIRS QUESTION BANK

Australis’s Willingness to Join the Grouping


For years, Australia has wanted to be party to the Malabar exercise. It
even agreed to join in as an observer in 2017.
Australia is a part of Quad group which includes India, USA, Japan and
Australia. However, Quad became dormant from 2008 to 2017.
While quad has been made operational once again since 2017, it only
took off in a significant way after a meeting between the countries’
foreign ministers in September 2019.
Also, tensions between Australia and China have been high since
Australia in April 2020 called for an investigation into the origins of
COVID-19.
China responded by reportedly drawing up a list of Australian exports
that could be subject to tariffs and stricter controls.
Why India Resisted The Inclusion Of Australia So Far?
India did not want to antagonise China. China has been uncomfortable
with the informal coalition of four democracies, which was first formed in
2004 to help nations in the Indo-Pacific after the tsunami and revived in
2017.
China views this maritime Quadrilateral as an Asian-NATO whose only aim
is to contain the rise of China.

What Has Changed Now?

Since 2007, India has tried to woo China. However, the situation has
changed now.
India has been facing a tense standoff with China after border clashes in
Galwan valley which resulted in the death of 20 Indian border troops and
an unknown number from the Chinese side.
This had changed India’s posture towards China. Now, India is under no
AVAIL OUR HELP ON INSTAGRAM/ TELEGRAM- DEFENCE MANIA
DOWLOAD “DEFENCE MANIA” APP
FOLLOW OUR YOUTUBE CHANNEL
CURRENT AFFAIRS QUESTION BANK

obligation to give priority to China’s wishes over its own strategic


requirements.
Positive Effects of Australia Joining the Exercise:
The decision to include Australia comes as Beijing and New Delhi are
caught up in their worst border tensions in four decades. It will give a
strong message to China.
The more India cooperates with the US, Japan and Australia, the more it
can improve its ability to counter China in the Indo-China border area.
Quad will not just be a symbol, but will also have more practical value.
There is a possibility that the Quad will expand as Quad+; Southeast Asian
countries like Vietnam, Indonesia, Philippines etc and other European
countries like the UK and France, too, will join the grouping.
Australia’s full induction into the Quad has implications for India’s
approach to the Indo-Pacific as well. So far, India was reluctant to term its
Indo-Pacific policy as a strategy, and mostly referred
to it as its “vision” of the Indo-Pacific.
However, this seems to be changing now. As the Quad starts to leverage
its hard power, and as India’s relations with China deteriorate, New Delhi
is likely to reshape its Indo-Pacific strategy alongside its partners.
Challenges
India faces a dilemma that differentiates its strategic concerns from those
of the US, Japan and Australia: India is alone among the four countries that
shares a land border with China.
As the cooperation among Quad countries increases, China might respond
by deploying an increasingly greater number of forces to the Chinese side
of the India-China border.
It should be remembered that, India and China are currently negotiating a
truce on the border in Eastern Ladakh. Formation of Quad might derail the
disengagement process at Ladakh.

AVAIL OUR HELP ON INSTAGRAM/ TELEGRAM- DEFENCE MANIA


DOWLOAD “DEFENCE MANIA” APP
FOLLOW OUR YOUTUBE CHANNEL
CURRENT AFFAIRS QUESTION BANK

Also, there is a possibility that China might respond through aggressive


posturing in the Eastern Indian Ocean. This will open up a new front in the
India-China conflict.
The US commitment, because of its military, economic and technological
power, is crucial. As of now, the US is focusing on its competition with
China and hence will not withdraw from this region any time soon. But if
there is any sign of US withdrawal, it will be nightmare for the other
countries.

Conclusion
The edifice of post 1988 India-China relationship was built on the premise
that China will not pose a security threat to India, and that it will be
sensitive towards India’s core interests. That has not happened. Hence,
India must pursue its strategic interests independently.
The invitation should not be a one-off measure to irritate China in the
backdrop of the border tension, but should be a permanent feature of
India’s strategic thinking.
Then only the vision of free, open and prosperous India-Pacific region will be
advanced and protected.

About Malabar Exercise:


Exercise Malabar is a trilateral naval exercise involving the United States,
Japan and India as permanent partners.

Originally begun in 1992 as a bilateral exercise between India and the


United States, Japan became a permanent partner in 2015.

AVAIL OUR HELP ON INSTAGRAM/ TELEGRAM- DEFENCE MANIA


DOWLOAD “DEFENCE MANIA” APP
FOLLOW OUR YOUTUBE CHANNEL
CURRENT AFFAIRS QUESTION BANK

43. Hul Divas is celebrated among tribals on June 30 every year. This is
celebrated in memory of
a) Birsa Munda
b) Sidho and Kanhu
c) Bhagoji Naik
d) Dirjinarain

Answer –
B
Explanati
on
Celebrated on June 30 in the memory of tribal leaders — Sidho and Kanhu
Murmu (Santhal Revolt).
On June 30, 1855 tribals under the leadership of Sidho and Kanhu
revolted at Bhognadih (a village in Sahebganj district of Jharkhand).

44. Who among the following persons have been appointed, on the
recommendation of Indian government, as vice-president of the Asian
Development Bank (ADB)?
a) Diwakar Gupta
b) Sunil Arora
c) Chandrachud Singh
d) Ashok Lavasa

45. Consider the following statements regarding Commonwealth Human Rights


Initiative (CHRI):
AVAIL OUR HELP ON INSTAGRAM/ TELEGRAM- DEFENCE MANIA
DOWLOAD “DEFENCE MANIA” APP
FOLLOW OUR YOUTUBE CHANNEL
CURRENT AFFAIRS QUESTION BANK

i. CHRI is an intergovernmental organization which aims to


promote human rights in the countries of the Commonwealth.
ii. It is headquartered at New
Delhi, India. Which of the above
statements is/are Correct?
a) 1 Only

b) 2 Only

c) Both 1 and 2
d) Neither 1 nor 2

Answer –
B
Explanatio
n
The Commonwealth Human Rights Initiative (CHRI) and an international anti-
slavery organisation Walk Free has released a report on Slavery. The report
was released on the occasion of World Day Against Trafficking in Persons.

Key Highlights Of The Report


Commonwealth countries accounts for about 40% of people living in
conditions of modern slavery in the world.
Commonwealth nations have failed to take proper actions to eradicate
modern slavery by 2030.
An estimated one in every 150 people in the Commonwealth is living in
conditions of modern slavery.
One-third of the Commonwealth countries had criminalised forced
marriage. 23 countries had not criminalised commercial sexual

AVAIL OUR HELP ON INSTAGRAM/ TELEGRAM- DEFENCE MANIA


DOWLOAD “DEFENCE MANIA” APP
FOLLOW OUR YOUTUBE CHANNEL
CURRENT AFFAIRS QUESTION BANK

exploitation of children.
Out of 54 countries, only four engage with business to investigate supply
chains, and all countries report gaps in victim assistance programs.

India Specific Observations


India, like all other Commonwealth countries in Asia, had not ratified the
International Labour Organisation’s 2011 Domestic Workers Convention
or the 2014 Forced Labour Protocol.

o The 2014 Forced Labour Protocol provides protection and


appropriate remedies, including compensation, to victims of forced
labour and to sanction the perpetrators of forced labour.
o It also obligates state parties to develop a national policy and plan of
action for the effective and sustained suppression of forced or
compulsory labour.
India accounted for one-third of all child brides in the world.
India has the weakest response on national coordination, with no
national coordinating body or
National Action Plan in place.”
Commonwealth Human Rights Initiative (CHRI)
It is an independent, non-profit, non-partisan, international non-
governmental organisation
working in the area of human rights.
HQ – New Delhi
The Commonwealth is a voluntary association of 54 independent and
equal sovereign states which came into existence in 1949.
Rwanda and Mozambique are two members which had no historical ties
with British empire.

46. Recently, Poland has expressed its intention to withdraw from the
AVAIL OUR HELP ON INSTAGRAM/ TELEGRAM- DEFENCE MANIA
DOWLOAD “DEFENCE MANIA” APP
FOLLOW OUR YOUTUBE CHANNEL
CURRENT AFFAIRS QUESTION BANK

Istanbul convention. The convention is related to


a) Preventing violence against women
b) Preventing Child Sexual Abuse
c) Elimination of Nuclear weapons
d) Fight against terrorism

Answer –
A
Explanati
on
Poland is to withdraw from Istanbul treaty aimed at preventing violence
against women.
Reasons Given By Poland
According to Poland, the Convention was harmful because it required
schools to teach children about gender.
The government also argued that the convention violated the rights of
parents and contains
elements of an ideological nature.
About The Treaty
The treaty, known as the Istanbul Convention, is intended to combat
violence against women in Europe.
Although the treaty does not address issues of gay rights, opponents have
claimed that the treaty promotes LGBT ideology and poses a threat to
Christian morality.
Ratification of the treaty has stalled in several European countries
including Bulgaria, the Czech Republic, Hungary, Latvia and Slovakia.
Russia, which is also a member of the Council of Europe, has not signed it.

AVAIL OUR HELP ON INSTAGRAM/ TELEGRAM- DEFENCE MANIA


DOWLOAD “DEFENCE MANIA” APP
FOLLOW OUR YOUTUBE CHANNEL
CURRENT AFFAIRS QUESTION BANK

The Blue Heart Campaign


Launched by UN
It is a global awareness raising initiative to fight human trafficking and its
impact on society.
The Blue Heart is increasingly recognized as the international symbol
against human trafficking, representing the sadness of those who are
trafficked while reminding us of the cold-heartedness of those who buy
and sell fellow human beings.

47. Who among the following has been nominated to be the vice-chairman of
the Security Platform of International Union of Railways (UIC):
a) Arun Kumar
b) Diwaker Sen Gupta
c) Ashok Lavasa
d) Piyush Goyal

Answer –
A
Explanati
on
Arun Kumar, Director General of the Railway Protection Force (RPF), has
been nominated to be the vice-chairman of the Security Platform of the
Paris-based Union Internationale Des Chemins/International Union of
Railways (UIC).
AVAIL OUR HELP ON INSTAGRAM/ TELEGRAM- DEFENCE MANIA
DOWLOAD “DEFENCE MANIA” APP
FOLLOW OUR YOUTUBE CHANNEL
CURRENT AFFAIRS QUESTION BANK

The security platform of UIC promotes the exchange of information and


experience among the security agencies of UIC members.
The UIC is the global platform for railway systems working on inter-
operability, developing common technical standards for railways across
the world and strengthening what is called “rail diplomacy”.

48. Arad and Carmel, recently seen in news, are


a) Submarines deployed by China in Indian Ocean
b) US’ warships deployed in South China Sea
c) Indian submarine and Aircraft carrier respectively
d) Israeli assault rifles

Answer –
D
Explanati
on
Two latest Israeli assault rifles — the Arad and the Carmel — are set to be
manufactured in India.
The assault weapons are to be produced under the ‘Make in India’
initiative.
A plant had been set up in Madhya Pradesh in 2017 by Israel Weapons
System (IWI) in a joint venture called PLR Systems

AVAIL OUR HELP ON INSTAGRAM/ TELEGRAM- DEFENCE MANIA


DOWLOAD “DEFENCE MANIA” APP
FOLLOW OUR YOUTUBE CHANNEL
CURRENT AFFAIRS QUESTION BANK

49. Consider the following statements:


i. Kargil Vijay Diwas is celebrated on July 27 every year.
ii. Raising Day of Central Reserve Police Force is
observed on 26 July. Which of the above statements is/are
Correct?
a) 1 Only

b) 2 Only

c) Both 1 and 2
d) Neither 1 nor 2

Answer – D
Explanation
21st Kargil Vijay Diwas was celebrated on 26 July to commemorate the
victory of the Indian soldiers over the infiltrating Pakistani troops.
The Kargil War fought between May-July of 1999 in the Kargil district of
Jammu and Kashmir along the Line of Control. Operation Vijay was
launched by the Indian army to throw back the intruders.
On July 26, 1999, India successfully regained command over all the high
outposts.
Raising Day of Central Reserve Police Force
The 82nd Raising Day of Central Reserve Police Force was celebrated on 27
July.
The CRPF came into existence as Crown Representative's Police on July 27,
1939. It became the Central Reserve Police Force (CRPF) on the enactment
AVAIL OUR HELP ON INSTAGRAM/ TELEGRAM- DEFENCE MANIA
DOWLOAD “DEFENCE MANIA” APP
FOLLOW OUR YOUTUBE CHANNEL
CURRENT AFFAIRS QUESTION BANK

of the CRPF Act on December 28, 1949. CRPF has the mandate to secure
internal security scenario of the country.

50. Consider the following statements regarding the Army Adventure Wing
(AAW):
i. AAW works under the Directorate General of Military Training.
ii. It is the apex body for the planning, conduct, coordination and
promotion of adventure activities in the Army.
Which of the above statements is/are correct?
a) 1 Only
b) 2 Only
c) Both 1 and 2
d) Neither 1 nor 2
Answer –
C
Explanati
on

The Indian Army has opened the Siachen Valley for the civilians. This
decision was taken amidst Line of Actual Control standoff between India
and China.
The Army Adventure Cell is to issue permits for

AVAIL OUR HELP ON INSTAGRAM/ TELEGRAM- DEFENCE MANIA


DOWLOAD “DEFENCE MANIA” APP
FOLLOW OUR YOUTUBE CHANNEL
CURRENT AFFAIRS QUESTION BANK

Siachen area. Army Adventure Wing:


It works under the Directorate General of Military Training. It is the apex
body for the planning, conduct, coordination and promotion of adventure
activities in the Army.
Presently AAW conducts training in 17 different adventure disciplines
categorised under three fields of Aero, Aqua and Land.
There are 25 state-of-the-art Army Adventure Training Centres in the Army
that impart specialised training and certification to all volunteers that
have the zeal and enthusiasm to pursue adventure activities.

51. Stickney, Shklovsky, Roche and Grildrig are craters of which of the following
planetary body
a) Moon
b) Mars

c) Phobos
d) Venus

Answer:
C
Explanati
on
● The Mars Colour Camera (MCC) onboard ISRO's Mars Orbiter Mission has
captured the image of

AVAIL OUR HELP ON INSTAGRAM/ TELEGRAM- DEFENCE MANIA


DOWLOAD “DEFENCE MANIA” APP
FOLLOW OUR YOUTUBE CHANNEL
CURRENT AFFAIRS QUESTION BANK

Phobos, the closest and biggest moon of Mars.


● Phobos is largely believed to be made up of carbonaceous chondrites.
● Stickney, the largest crater on Phobos along with the other craters
(Shklovsky, Roche & Grildrig) are also seen in this image.
Mars Orbiter Mission (MOM)
● Launched from Sriharikota (Andhra Pradesh) on 5th Nov’ 2013 by ISRO, the
MOM or the
Mangalyaan is a space probe, orbiting Mars since 24 September 2014.
● It is India's first interplanetary mission and it made ISRO the fourth space
agency to reach Mars, after Roscosmos, NASA, and the European Space
Agency.
● It made India the first Asian nation to reach Martian orbit and the first
nation in the world to do so on its maiden attempt.
● The mission was initially meant to last six months, but subsequently
ISRO had said it had enough fuel for it to last "many years."
● The Rs 450-crore MOM mission aims at studying the Martian surface and
mineral composition as well as scan its atmosphere for methane (an
indicator of life on Mars).
● The Mars Orbiter has five scientific instruments - Lyman Alpha
Photometer (LAP), Methane Sensor for Mars (MSM), Mars Exospheric
Neutral Composition Analyser (MENCA), Mars Colour Camera (MCC) and
Thermal Infrared Imaging Spectrometer (TIS).
Chondrite
● A chondrite is a stony (non-metallic) meteorite
that has not been modified, by either melting or
differentiation of the parent body.
AVAIL OUR HELP ON INSTAGRAM/ TELEGRAM- DEFENCE MANIA
DOWLOAD “DEFENCE MANIA” APP
FOLLOW OUR YOUTUBE CHANNEL
CURRENT AFFAIRS QUESTION BANK

● They are formed when various types of dust and


small grains in the early Solar System accreted
to form primitive asteroids.
● Estimates for their contribution to the total
meteorite population vary between 85.7% and
86.2%.
● Their study provides important clues for
understanding the origin and age of the Solar
System, the synthesis of organic compounds,
the origin of life and the presence of water on
Earth.

52. Weibo Diplomacy, sometimes seen in news is related with


a) Foreign embassies to direct communicate with people of China
b) Chinese policy to increase its presence in African Region
c) Group of Countries against China’s South China Sea aggression
d) US policy to support Hong Kong independence

Answer: A
Explanati
on
● After the government banned 59 Chinese apps, Prime Minister Narendra
Modi has decided to quit the Chinese microblogging website Sina Weibo,
which he had joined a few years ago.
● This marks a sudden end to Mr. Modi’s “Weibo diplomacy” in China that
AVAIL OUR HELP ON INSTAGRAM/ TELEGRAM- DEFENCE MANIA
DOWLOAD “DEFENCE MANIA” APP
FOLLOW OUR YOUTUBE CHANNEL
CURRENT AFFAIRS QUESTION BANK

was launched with much fanfare in 2015 as a means to directly


communicate with the people of China before his first visit there as Prime
Minister.
● Weibo diplomacy: Foreign embassies communicating on Chinese social
media

53. Consider the following sentences about G4 Flu Virus


i. G4 flu virus is a newly emerged strain of influenza virus that is
infecting Chinese chicken and that has the potential of
triggering a pandemic.
ii. It is reported that the new strain (G4) has descended from the
H5N7 strain that was responsible for the 2009 swine flu
pandemic.
iii. It is also found that the G4 strain has the capability of binding to
human-type receptors.

AVAIL OUR HELP ON INSTAGRAM/ TELEGRAM- DEFENCE MANIA


DOWLOAD “DEFENCE MANIA” APP
FOLLOW OUR YOUTUBE CHANNEL
CURRENT AFFAIRS QUESTION BANK

Which of the above statements is/are incorrect


a) 1 and 2 only
b) 2 and 3 only
c) 3 only
d) 1, 2 and 3 only

Answer:
A
Explanati
on:
● In a new research, scientists from China have identified a “recently
emerged” strain of influenza virus,
G4 flu virus, that is infecting Chinese pigs and that has the potential of
triggering a pandemic.
● Scientists report that the new strain (G4) has descended from the H1N1
strain that was responsible for the 2009 swine flu pandemic.
● They also found that the G4 strain has the capability of binding to human-
type receptors (like, the SARS-CoV-2 virus binds to ACE2 receptors in
humans), was able to copy itself in human airway epithelial cells, and it
showed effective infectivity and aerosol transmission in ferrets.

54. Which of the following disease is also known as Black death in Middle age
a) Measles
b) Kawasaki disease
c) Kala Azar
d) Bubonic Plague

AVAIL OUR HELP ON INSTAGRAM/ TELEGRAM- DEFENCE MANIA


DOWLOAD “DEFENCE MANIA” APP
FOLLOW OUR YOUTUBE CHANNEL
CURRENT AFFAIRS QUESTION BANK

Answer: D
Explanation
● A suspected case of bubonic plague was reported from Bayannur, a city in
northern China.
● The bubonic plague, known as the “Black Death” in the Middle Ages, is a
highly infectious and often fatal disease that is spread mostly by rodents.
It is a zoonotic disease, which means it can jump from animals to humans.
● The plague is estimated to have a basic reproduction number, or R0,
between 5 and 7. This means that one infected person can spread the
disease to 5-7 people. The ongoing Covid-19 pandemic’s R0 is between 1
and 2.
● Some of the symptoms of the disease include swollen lymph nodes which
can be as large as chicken eggs, fever, chills, coughs, fatigue, muscle ache,
etc. The swollen lymph nodes are called buboes, which is where the name
‘Bubonic’ comes from.
Plague
● Plague is an infectious disease caused by the bacteria Yersinia pestis,
usually found in small mammals and their fleas. The disease is transmitted
between animals via their fleas and, as it is a zoonotic bacteria, it can also
transmit from animals to humans.
● Humans can be contaminated by the bite of infected fleas, through direct
contact with infected materials, or by inhalation. Plague can be a very
severe disease in people with a case-fatality ratio of 30% - 100% if left
untreated.
● Although plague has been responsible for widespread pandemics
throughout history, today it can be easily treated with antibiotics and the
use of standard preventative measures.
● Plague is found on all continents except Oceania but most human cases
AVAIL OUR HELP ON INSTAGRAM/ TELEGRAM- DEFENCE MANIA
DOWLOAD “DEFENCE MANIA” APP
FOLLOW OUR YOUTUBE CHANNEL
CURRENT AFFAIRS QUESTION BANK

since the 1990s have occurred in Africa. Democratic Republic of Congo,


Madagascar and Peru are the three most endemic countries.
Kawasaki Disease
● In India and elsewhere, a new illness, with some symptoms common with
the rare Kawasaki disease, such as rashes and inflammation, has been
affecting children with Covid-19.
● Kawasaki disease is a syndrome of unknown cause that results in a fever
and mainly affects children under 5 years of age. It is a form of vasculitis,
where blood vessels become inflamed throughout the body.
● It is a rare disease and does not spread between people.
● The disease derives its name from a Japanese paediatrician, Tomisaku
Kawasaki, who reported the first case in 1961. The doctor, 95, died on June
5 this year in Tokyo.

55. Consider the following sentences about Science and Engineering Research
Board (SERB)
i. Science and Engineering Research Board (SERB) is an attached
body under the Department of Science and Technology,
Government of India.
ii. SERB launched ABHYAAS programme which aims to boost
research and development in the country by enabling and
grooming potential PG/PhD students by means of developing
their research skills in selected areas across different disciplines
or fields.
Which of the above sentences is/are correct?
a) 1 only
b) 2 only

c) Both 1 and 2
AVAIL OUR HELP ON INSTAGRAM/ TELEGRAM- DEFENCE MANIA
DOWLOAD “DEFENCE MANIA” APP
FOLLOW OUR YOUTUBE CHANNEL
CURRENT AFFAIRS QUESTION BANK

d) Neither 1 nor 2

Answer:
B
Explanati
on
● The Accelerate Vigyan (AV) scheme has been recently launched by the
Science and Engineering Research Board (SERB) to provide a single
platform for research internships, capacity building programs, and
workshops across the country.
● The primary objective of this inter-ministerial scheme is to give more
thrust on encouraging high- end scientific research and preparing scientific
manpower, which can lead to research careers and knowledge-based
economy.

● An Inter-Ministerial Overseeing Committee (IMOC) involving all the


scientific ministries/departments and a few others has been constituted
for the purpose of supporting SERB in implementing the AV scheme in a
successful manner.
● The vision is to expand the research base, with three broad goals, namely,
○ consolidation / aggregation of all scientific programs,
○ initiating high-end orientation workshops, and
○ creating opportunities for research internships for those who do not
have access to such resources / facilities.
● Components:
i) ‘ABHYAAS’ programme: It is an attempt to boost research and
AVAIL OUR HELP ON INSTAGRAM/ TELEGRAM- DEFENCE MANIA
DOWLOAD “DEFENCE MANIA” APP
FOLLOW OUR YOUTUBE CHANNEL
CURRENT AFFAIRS QUESTION BANK

development in the country by enabling and grooming potential


PG/PhD students by means of developing their research skills in selected
areas across different disciplines or fields. It has two components:
o High-End Workshops (‘KARYASHALA’) and Research Internships
(‘VRITIKA’). This is especially important for those researchers who
have limited opportunities to access such learning capacities /
facilities / infrastructure.
ii) SAMMOHAN: It has been sub-divided into ‘SAYONJIKA’ and
‘SANGOSHTI’.
o SAYONJIKA is an open-ended program to catalogue the capacity
building activities in science and technology supported by all
government funding agencies in the country.

Science and Engineering Research Board (SERB)


● Science and Engineering Research Board (SERB) is a statutory body under the Department of
Science and Technology, Government of India established through the Science and
Engineering Research Board Act, 2008.
● It aims to support basic research in emerging areas of Science & Engineering. It also aims to
build up best management systems which would match the best global practices in the area of
promotion and funding of basic research.

o SANGOSHTI is a pre-existing program of SERB.

56. BlackRock, recently in news, is related with


a) Name given to the newly found exoplanet with Earth-like geological
structure.
b) Rock found on Antarctica that contains remains of million-year-old algae
c) A newly found android malware
d) Name given to mountains on Moon
Answer: C

AVAIL OUR HELP ON INSTAGRAM/ TELEGRAM- DEFENCE MANIA


DOWLOAD “DEFENCE MANIA” APP
FOLLOW OUR YOUTUBE CHANNEL
CURRENT AFFAIRS QUESTION BANK

Explanati
on:
● Security firm ThreatFabric has alerted about a new malware, called
BlackRock, which can steal information like passwords and credit card
information from about 377 smartphone applications. Since these are very
popular apps, the threat posed by the BlackRock Android malware is quite
high.
● BlackRock isn’t a new strain of malware. It is based on the leaked source code
of the Xeres malware,
which itself derived from malware called LokiBot.
● BlackRock works like most Android malware. Once installed on a phone, it
monitors the targeted app. When the user enters the login and/or credit
card details, the malware sends the information to a server. BlackRock
uses the phone’s Accessibility feature, and then uses an Android DPC
(device policy controller) to provide access to other permissions.
● The only big difference between BlackRock and other Android banking
trojans is that it can target more apps than previous malwares including
Amazon, Facebook, Gmail, Tinder, Uber, Twitter, Snapchat, PayPal etc.
● But BlackRock isn’t limited to online banking apps and targets general
purpose apps also across various categories of Books & Reference,
Business, Communication, Dating, Entertainment, Lifestyle, Music &
Audio, News & Magazine, Tools, and Video Players & Editors.
● The malware can be used to send and steal SMS messages, hide
notifications, keylogging, AV detection, and much more.

AVAIL OUR HELP ON INSTAGRAM/ TELEGRAM- DEFENCE MANIA


DOWLOAD “DEFENCE MANIA” APP
FOLLOW OUR YOUTUBE CHANNEL
CURRENT AFFAIRS QUESTION BANK

Banking Trojans
● A trojan is any type of malicious program disguised as a legitimate one. Often, they are
designed to steal sensitive information (login credentials, account numbers, financial information,
credit card information, and the like) from users.
● Banking trojans are a specific kind of trojan malware. Once installed onto a client machine,
banking trojans use a variety of techniques to create botnets, steal credentials, inject malicious
code into browsers, or steal money.

57. 2020 ND, recently in news, is related with


a) A potentially dangerous Asteroid
b) New strain of Novel Coronavirus
c) A New species of medical plant found in Andaman
d) None of the above

Explanation
● NASA has issued a warning of a huge “Asteroid 2020 ND” that is expected to
move past Earth.
● The asteroid, about 170 metres-long, will be as close as 0.034 astronomical
units (5,086,328 kilometres) to

● the earth, and is travelling at a speed of 48,000 kilometres per hour. Its
distance from Earth has placed it in the “potentially dangerous” category.
Asteroids
● Asteroids, sometimes called minor planets, are rocky remnants left over
from the early formation of our solar system about 4.6 billion years ago.
● Most of this ancient space rubble can be found orbiting the sun between
Mars and Jupiter within the
main asteroid belt.
● Asteroids range in size from Vesta—the largest at about 329 miles (530
AVAIL OUR HELP ON INSTAGRAM/ TELEGRAM- DEFENCE MANIA
DOWLOAD “DEFENCE MANIA” APP
FOLLOW OUR YOUTUBE CHANNEL
CURRENT AFFAIRS QUESTION BANK

kilometers) in diameter - to bodies that are less than 33 feet (10 meters)
across. The total mass of all the asteroids combined is less than that of
Earth's Moon.
Potentially Hazardous Asteroids
● Potentially Hazardous Asteroids (PHAs) are currently defined based on
parameters that measure the asteroid’s potential to make threatening
close approaches to the Earth. Specifically, all asteroids with a minimum
orbit intersection distance (MOID) of 0.05 AU or less are considered PHAs.
● NASA classifies objects like these as ‘near-Earth objects’ (NEOs) as they get
nudged by other planets’ gravitational attraction resulting in their
proximity to our solar system.
Near-Earth objects (NEOs)
● NEOs are comets and asteroids nudged by the gravitational attraction of
nearby planets into orbits which allows them to enter the Earth’s
neighbourhood.
● These objects are composed mostly of water ice with embedded dust
particles, and occasionally approach close to the Earth as they orbit the
Sun.
● NASA’s Center for Near-Earth Object Study (CNEOS) determines the times
and distances of these objects as and when their approach to the Earth is
close.
● NASA’s Near-Earth Object Observations Program finds, tracks and
characterises over 90% of the predicted number of NEOs that are 140
metre or larger, which according to the space agency are of “the greatest
concern” due to the level of devastation that their impact is capable of
causing.
Deflecting Asteroids
● Over the years, scientists have suggested different ways to ward off such
threats, such as blowing up the asteroid before it reaches Earth, or

AVAIL OUR HELP ON INSTAGRAM/ TELEGRAM- DEFENCE MANIA


DOWLOAD “DEFENCE MANIA” APP
FOLLOW OUR YOUTUBE CHANNEL
CURRENT AFFAIRS QUESTION BANK

deflecting it off its Earth-bound course by hitting it with a spacecraft.


● Asteroid Impact and Deflection Assessment (AIDA): It is the most drastic
measure undertaken so far and includes NASA’s Double Asteroid
Redirection Test (DART) Mission and the European Space Agency’s (ESA)
Hera.
● The mission’s target is Didymos, a binary near-Earth asteroid, that could
pose the most likely significant threat to Earth.

● It aims to slam into the smaller asteroid of the Didymos system at around 6 km per second in 2022.
Hera
● Hera is a project of ESA which is scheduled to launch in 2024 and will arrive at the Didymos
system in 2027.
● It aims to measure the impact crater produced by the DART collision and study the change in
the asteroid’s orbital trajectory.

58. Which of the following pair(s) is/are


correctly matched Spacecraft/Mission
Country/Region
a. Perseverance : United State of America
b. InSight : European Union
c. Hope : Saudi Arabia
d. Tianwen : China
Select the correct answer using the codes given below.
a) 1 and 4 only
b) 1, 2 and 4 only
c) 1, 3 and 4 only
d) 1, 2, 3 and 4
AVAIL OUR HELP ON INSTAGRAM/ TELEGRAM- DEFENCE MANIA
DOWLOAD “DEFENCE MANIA” APP
FOLLOW OUR YOUTUBE CHANNEL
CURRENT AFFAIRS QUESTION BANK

Answer: A
Explanati
on:
Hope was the first of three space missions sent toward Mars during the July
2020 Mars launch window, with missions also launched by the national space
agencies of China (Tianwen-1) and the U.S. (Mars 2020 or Perseverance). All
three are expected to arrive at Mars in February 2021. This Mars-launching
season - which occurs every 26 months when Earth and Mars are at their
closest - is especially busy.

Hope: Emirates Mars Mission


● The Emirates Mars Mission is a United Arab Emirates Space Agency
uncrewed space exploration mission to Mars. The Hope (Al Amal) orbiter
was launched on 19 July 2020, marking the first planetary science mission
led by an Arab country.
● It was launched from Tanegashima, LP-1 (Japan) by a Japanese H-IIA rocket,
● In just six years, the UAE has built an impressive space program from
scratch. The mission design, development, and operations are led by the
Mohammed bin Rashid Space Centre (MBRSC).
● It is a joint project between UAE and the University of Colorado Boulder,
University of California, Berkeley, and Arizona State University, and was
assembled at the University of Colorado (US).
● The spacecraft orbiter will orbit the Red Planet and study the Martian
atmosphere and climate. It will study

daily and seasonal weather cycles, weather events in the lower


atmosphere such as dust storms, and how the weather varies in different

AVAIL OUR HELP ON INSTAGRAM/ TELEGRAM- DEFENCE MANIA


DOWLOAD “DEFENCE MANIA” APP
FOLLOW OUR YOUTUBE CHANNEL
CURRENT AFFAIRS QUESTION BANK

regions of the planet. It will give planetary scientists their first global view
of Martian weather at all times of day.
● It will also attempt to find out why it is losing hydrogen and oxygen into
space and other possible reasons behind its drastic climate changes.

TIANWEN-1 (TW-1)
● China launched its most ambitious Mars mission, Tianwen-1, on 23rd July
2020 on a Long March-5 carrier rocket (nicknamed "Fat 5" because of its
bulky shape) from Hainan Island, a resort province off the south coast of
the mainland.
● Tianwen-1 (meaning quest for heavenly truth) contains both an orbiter
and a rover and will take seven months to reach Mars (February 2021) and
look for a landing site on Utopia Planitia - a plain where NASA has detected
possible evidence of underground ice. The landing would then be
attempted in April or May.
● If all goes well, the 240-kilogram (530-pound) golf cart-sized, solar-
powered rover is expected to
operate for about three months, and the orbiter for two years.
● Though small compared to America's hulking, car-sized 1,025-kilogram
(2,260-pound) Perseverance, it's almost twice as big as the two rovers
China has sent to the moon in 2013 and 2019.
● Its objectives are to search for evidence of underground water, if it's
present, as well as evidence of possible ancient life. It will also assess the
planet's environment.
● This isn't China's first attempt at Mars. In 2011, a Chinese orbiter
accompanying a Russian mission was lost when the spacecraft failed to get
out of Earth's orbit after launching from Kazakhstan, eventually burning
up in the atmosphere.

NASA: Perseverance Rover


AVAIL OUR HELP ON INSTAGRAM/ TELEGRAM- DEFENCE MANIA
DOWLOAD “DEFENCE MANIA” APP
FOLLOW OUR YOUTUBE CHANNEL
CURRENT AFFAIRS QUESTION BANK

● Nasa has successfully launched its next-generation Perseverance rover on


a seven-month journey to Mars on 30 July 2020 in search of evidence of
ancient microbial life on the red planet. This will advance NASA's quest to
explore the past habitability of Mars.
● The car-sized robot will attempt to land on the Jezero crater – which may
have been a lake more than 3.5bn years ago – where it will gather
information about Mars’s geology, atmosphere and environmental
conditions.
● Perseverance will carry a mini helicopter, Ingenuity, that will attempt the
first powered flight on another planet, and also test technologies to help
pave the way for future human exploration of Mars. These include testing
a method for producing oxygen from the Martian atmosphere, identifying
other resources (such as subsurface water), improving landing techniques,
and characterizing weather, dust, and other potential environmental
conditions that could affect future astronauts living and working on Mars.
● The Perseverance rover has four science objectives that support the Mars
Exploration Program's science goals:
○ Identify past environments capable of supporting microbial life
○ Seek signs of possible past microbial life in those habitable
environments, particularly in special rocks

○ known to preserve signs of life over time


○ Collect core rock and "soil" samples and store them on the Martian
surface
○ Test oxygen production from the Martian atmosphere

Mars Exploration Programme


● The Mars Exploration Program (MEP), initially called the Mars Surveyor Program, was announced
by NASA in 1994.

AVAIL OUR HELP ON INSTAGRAM/ TELEGRAM- DEFENCE MANIA


DOWLOAD “DEFENCE MANIA” APP
FOLLOW OUR YOUTUBE CHANNEL
CURRENT AFFAIRS QUESTION BANK

● It is a science-driven, technology-enabled study of Mars as a planetary


system in order to understand:
a) the formation and early evolution of Mars as a planet
b) the history of geological and climate processes that have shaped Mars
through time
c) the potential for Mars to have hosted life (its “biological potential”)
d) the future exploration of Mars by humans, and
e) how Mars compares to and contrasts with Earth.

● Landing on Mars is notoriously difficult. Only the US has successfully


landed a spacecraft on Martian soil, doing it eight times since 1976.
● NASA's InSight and Curiosity rovers still operate today. Six other
spacecraft are exploring Mars from orbit: three American, two
European and one from India.

Interplanetary Contamination
● As ambitious space missions are proliferating, along with advances in
commercial space flight,
astrobiologists have expressed concerns about possible ‘interplanetary
contamination’.
● Interplanetary contamination refers to biological contamination of a
planetary body by a space probe or spacecraft, either deliberate or
unintentional.
● Such contamination are of two types
AVAIL OUR HELP ON INSTAGRAM/ TELEGRAM- DEFENCE MANIA
DOWLOAD “DEFENCE MANIA” APP
FOLLOW OUR YOUTUBE CHANNEL
CURRENT AFFAIRS QUESTION BANK

1. Forward Contamination: The transport of Earth-based microbes to other


celestial bodies
o In the past, space missions have established physical contact with
astronomical bodies such as comets and asteroids, and crewed
missions have landed on the Moon. However, since these bodies
are known to be hostile to life, the possibility of their forward
contamination has not been a pressing issue.
o In the case of Mars, however, space missions have already
discovered the possible presence of liquid water on the planet,
either in the subsurface today or at some point in its past, and are
now actively looking for signs of life.
o Astrobiologists say that if there is a chance that Mars has life, even
in its most primitive form, there is an ethical obligation on
humanity to ensure that microbes from Earth do not disturb a
possible Martian biosphere.
Back Contamination: The transfer of extraterrestrial organisms (if they
exist) into the Earth’s biosphere.
o NASA also plans for a Mars sample-return mission, which would
bring samples of the Red Planet back to Earth, possibly by 2031.
o The suggestion that Martian microbes (if they exist) would infect
human beings–causing a catastrophe like the current pandemic– is
highly unlikely, but possibility can’t be ruled out.

59. Consider the following statements about Malaria


i. Malaria is a mosquito-borne infectious disease that affects
humans and other animals.
ii. It is caused by single-celled microorganisms of the Plasmodium
AVAIL OUR HELP ON INSTAGRAM/ TELEGRAM- DEFENCE MANIA
DOWLOAD “DEFENCE MANIA” APP
FOLLOW OUR YOUTUBE CHANNEL
CURRENT AFFAIRS QUESTION BANK

group.
iii. The disease is most commonly spread by an infected female
Anopheles mosquito.

iv. Dichlorodiphenyltrichloroethane (DDT) is used to control


Malaria.
v. Hindustan Insecticides Limited (HIL, India) is the sole
manufacturer of DDT globally. Which of the above sentences is/are
correct?
b) 1, 2, 3 and 4 only

c) 2, 3 and 5 only

d) 1, 3, 4 and 5 only
e) 1, 2, 3, 4 and 5

Answer: D
Explanati
on:
● Hindustan Insecticides Limited (HIL, India), a PSU under the Ministry of
Chemicals and Fertilizers, supplied 20.60 Metric tonne of DDT to South
Africa for their Malaria control program. HIL (India) is the sole
manufacturer of DDT globally. The company was incorporated in 1954 to
manufacture and supply DDT to the Government of India’s Ministry of
Health and Family Welfare for malaria control programmes.
● Malaria is a mosquito-borne infectious disease that affects humans and
other animals.
● It is caused by single-celled microorganisms of the Plasmodium (parasite)
group. The disease is most commonly spread by an infected female
Anopheles mosquito.

AVAIL OUR HELP ON INSTAGRAM/ TELEGRAM- DEFENCE MANIA


DOWLOAD “DEFENCE MANIA” APP
FOLLOW OUR YOUTUBE CHANNEL
CURRENT AFFAIRS QUESTION BANK

● Dichlorodiphenyltrichloroethane (DDT), is a colourless, tasteless, and


almost odorless crystalline chemical compound, an organochlorine.
Originally developed as an insecticide, it became infamous for its
environmental impacts.

60. What is Miyawaki Method, recently in news?


a) Unique method of forest creation
b) Tiger monitoring method
c) Method to prevent phishing attacks
d) Unique method of Aquacultur
Answer: A
Explanati
on:
Union Environment Minister inaugurated a unique urban forest at the
office of the Comptroller and
Auditor General of India (CAG) in New Delhi. The Miyawaki method of
forest creation has been used to create the urban forest.

The technique was pioneered by Japanese botanist, Akira Miyawaki, and


is believed to ensure 10 times faster plant growth, which results in
plantation, that is 30 times denser than usual.
Using this, it’s possible to grow a variety of native species in as little a space
as 600 sq.ft.
The Miyawaki method could help in reducing the temperature by as much
as 14 degree & increase the moisture by more than 40%.
Recently Yadadri method of planting, a modified version of Miyawaki
Method, has been employed in the annual afforestation program of
Telangana state government.
AVAIL OUR HELP ON INSTAGRAM/ TELEGRAM- DEFENCE MANIA
DOWLOAD “DEFENCE MANIA” APP
FOLLOW OUR YOUTUBE CHANNEL
CURRENT AFFAIRS QUESTION BANK

61. Consider the following statements about Central Zoo Authority (CZA)
i. It is a statutory body under the Ministry of Environment, Forest
and Climate Change.
ii. It is chaired by Union Environment Minister.

Which of the statements given above is/are correct?


a) 1 Only
b) 2 Only
c) Both 1 and 2
d) Neither 1 nor 2

Answer: C
Explanati
on:
The Environment Ministry has reconstituted the Central Zoo Authority
(CZA) to include an expert from the School of Planning and Architecture,
Delhi, and a molecular biologist.
The Central Zoo Authority (CZA) is a statutory body under the Ministry of
Environment, Forest and Climate Change. It was constituted in 1992
under the Wildlife (Protection) Act, 1972.
It is chaired by the Environment Minister and tasked with regulating zoos
across the country. Apart from the chairman, it consists of 10 members
and a member-secretary.
The main objective of the authority is to complement the national effort
in the conservation of the rich biodiversity of the country, particularly the
fauna as per the National Zoo Policy, 1998.

AVAIL OUR HELP ON INSTAGRAM/ TELEGRAM- DEFENCE MANIA


DOWLOAD “DEFENCE MANIA” APP
FOLLOW OUR YOUTUBE CHANNEL
CURRENT AFFAIRS QUESTION BANK

62. Brahmapuram Solid Waste Processing Plant recently in news was located
in which among the following states?
a) Kerala

b) Karnataka

c) Maharashtra

d) Andhra Pradesh

Answer:
Explanati
on:
National Green Tribunal (NGT) has said that the Chairman and Member
Secretary of the Kerala State Pollution Control Board (KSPCB) will be held
liable if they fail to initiate prosecution and recover compensation from
those responsible for the unscientific handling of waste at the dumping
site of the Kochi Corporation at Brahmapuram.
The tribunal’s order came in the case related to the non-compliance with
Solid Waste Management Rules, 2016 (SWM Rules, 2016) at the now
defunct Brahmapuram solid waste processing plant on the outskirts of
Kochi city resulting in environment pollution and impacting public health.
The tribunal expressed doubts on whether the leachate generated at the
dumping yard could be treated in an ordinary septage treatment plant as
it contained heavy metals. This observation was made in reply to the
board’s report that the construction of a new leachate collection tank of
one-lakh- litre collection capacity was nearing completion.

63. Consider the following statements about Green-Ag Project


i. It aims to reduce emissions from agriculture and ensure
sustainable agricultural practices.
AVAIL OUR HELP ON INSTAGRAM/ TELEGRAM- DEFENCE MANIA
DOWLOAD “DEFENCE MANIA” APP
FOLLOW OUR YOUTUBE CHANNEL
CURRENT AFFAIRS QUESTION BANK

ii. The Green-Ag Project is funded by the


World Bank. Which of the statements given
above is/are correct?
a) 1 Only

b) 2 Only

c) Both 1 and 2
d) Neither 1 nor 2

Answer: A
Explanati
on:
The Union government launched the “Green – Ag: Transforming Indian
Agriculture for global environment benefits and the conservation of critical
biodiversity and forest landscapes” project in Mizoram.
About Green-Ag Project
Aim: To reduce emissions from agriculture and ensure sustainable
agricultural practices.
Objectives:
o Achieve multiple global environmental benefits in at least 1.8 million
hectares (ha) of land in five landscapes, with mixed land use systems.
o Bring at least 104,070 ha of farms under sustainable land and water
management.
o Ensure 49 million Carbon dioxide equivalent (CO2 eq) sequestered or
reduced through sustainable land use and agricultural practices.
Coverage: The project will be implemented in five states - Mizoram,
Rajasthan, Madhya Pradesh, Odisha and Uttarakhand
Funding & Implementation: The Green-Ag Project is funded by the Global
Environment Facility, while the Department of Agriculture, Cooperation,
AVAIL OUR HELP ON INSTAGRAM/ TELEGRAM- DEFENCE MANIA
DOWLOAD “DEFENCE MANIA” APP
FOLLOW OUR YOUTUBE CHANNEL
CURRENT AFFAIRS QUESTION BANK

and Farmers’ Welfare (DAC&FW) is the national executing agency. Other


key players involved in its implementation are Food and Agricultre
Organization (FAO) and the Union Ministry of Environment, Forest and
Climate Change (MoEF&CC).
The pilot project is supposed to end on March 31, 2026, in all states.

With Chinese railway hardly ever sharing any data, Indian Railways is a
significant member of the UIC as the largest mover of passengers and
freight after China.

The new testing plan also includes substantial number of rapid antibody-
based blood tests as part of containment strategy in high-risk areas and
hotspots. This mainly includes clusters and large migrations, gatherings or
evacuee’s centres.
To trace how infections like the novel coronavirus have spread so far, it is
important to detect people who contracted the disease in the past and
have recovered. This is what serological tests seek to determine.
Unlike genetic tests, which look for RNA in swab samples, serological tests
work on antibodies in blood samples. Hence, they are also called antibody
tests.
Antibodies, or protective proteins produced by the immune system to
neutralise pathogens such as bacteria and viruses, are present in one’s
bloodstream for a considerable period of time after the infection has gone.
To disable a pathogen, the antibody latches to a unique protein molecule
on pathogen’s surface, called an antigen. Serological tests use antigen
molecules to detect the presence of antibodies relevant to the infection.
Generally, a blood sample is placed in a test tube that is lined with antigens
on the inside. If the relevant antibodies are present, they latch on to the
antigens. Such tests are relatively inexpensive, and can display results
AVAIL OUR HELP ON INSTAGRAM/ TELEGRAM- DEFENCE MANIA
DOWLOAD “DEFENCE MANIA” APP
FOLLOW OUR YOUTUBE CHANNEL
CURRENT AFFAIRS QUESTION BANK

within a few minutes.


It is important to note, however, that the PCR test is capable of identifying
infection at an earlier stage. Only after the antibodies have developed,
which takes several days, can the serological test come in. And even for
serological tests, the positives will have to go through the PCR filter.
Conventional RT-PCR test is a direct evidence but antibody test can only
indicate the presence of the virus, therefore, it is indirect evidence.
Testing Strategy in India
Experts have criticised India’s testing strategy and raised concerns about
the sample size of testing being too little to rule out community
transmission. However, the government has maintained that testing had
to be process driven and could not merely be a “confidence building
exercise”.
The health ministry has maintained that it is framing its testing strategy in
response to the developing situation, keeping in mind the need to test
those who require to be scanned and to ensure wasteful testing does not
clog the system.
After focusing on foreign travellers and their contacts who showed
symptoms, testing was widened to include all serious patients who
reported flu-like illnesses.
The increase in suspected cases due to the migration of workers, Tablighi
Jamaat outbreak and the need to keep an eye on clusters has seen the
government expand its testing coverage significantly.

64. Considerthe following statements about the Huntington Disease:


a. Huntington disease is a genetic disease.
b. Huntington disease affects the brain.
c. There is no cure for Huntington disease
at present. Which of the statements given

AVAIL OUR HELP ON INSTAGRAM/ TELEGRAM- DEFENCE MANIA


DOWLOAD “DEFENCE MANIA” APP
FOLLOW OUR YOUTUBE CHANNEL
CURRENT AFFAIRS QUESTION BANK

above is/are correct?


a) 1 and 2 only

b) 2 and 3 only

c) 1 and 3 only

d) 1, 2 and 3 only

Answer: D
Explanatio
n:
Scientists from National Centre for Cell Science (NCCS) in Pune have gained
insights into the Huntington disease by studying the HTT gene in fruit flies.
Huntington Disease
Huntington disease (HD) is a progressive genetic disorder affecting the brain
that causes uncontrolled movements, impaired coordination of balance
and movement, a decline in cognitive abilities, difficulty in concentrating
and memory lapses, mood swings and personality changes.
It is an autosomal dominant genetic disorder, which means that even if one
parent carries the defective Huntington’s gene, their offspring has a 50:50
chance of inheriting the disease.
Every individual with the gene for the disease will eventually develop the
disease and there is no treatment or cure for Huntington’s at present.
It is caused by a mutation in a gene called HTT. The HTT genes are involved
in the production of a protein called huntingtin. They provide the
instruction for making the protein.
When the genes mutate, they provide faulty instructions leading to
production of abnormal huntingtin proteins and these form into clumps.
AVAIL OUR HELP ON INSTAGRAM/ TELEGRAM- DEFENCE MANIA
DOWLOAD “DEFENCE MANIA” APP
FOLLOW OUR YOUTUBE CHANNEL
CURRENT AFFAIRS QUESTION BANK

The clumps disrupt the normal functioning of the brain cells, which
eventually leads to death of neurons in the brain, resulting in Huntington
disease.

Findings Of The Study


While it is known that the clumps formed by the abnormal huntingtin
protein disrupt several cellular processes, it is not known whether they
also influence the key process in the formation of other proteins in the cell.

Scientist from NCCS have observed that the pathogenic Huntingtin protein
causes a decrease in the overall protein production in cells and that the
Huntingtin clumps collect together (sequester) molecules of another
protein called Orb2, which is involved in the process of protein formation.
In humans, a family of proteins called CPEB is equivalent to the Orb2
protein in fruit flies. The scientists conducted further studies and found
that CPEB proteins are also sequestered by the pathogenic Huntingtin
clumps, similar to the Orb2 protein molecules.
This suggests that the insights gained through the studies carried out by
this group in fruit flies are relevant to and valuable in understanding
Huntington Disease in humans. The findings are expected to pave the way
for further exploration to understand this disease better.

65. Consider the following statements about National Cadet Corps (NCC):
i. NCC is the world’s largest uniformed youth organisation.
ii. NCC is a Tri-Services organisation, comprising the Army, Navy
and Air Wings. Which of the statements given above is/are
correct?
a) 1 only

b) 2 only

AVAIL OUR HELP ON INSTAGRAM/ TELEGRAM- DEFENCE MANIA


DOWLOAD “DEFENCE MANIA” APP
FOLLOW OUR YOUTUBE CHANNEL
CURRENT AFFAIRS QUESTION BANK

c) Both 1 and 2
d) Neither 1 nor 2

Answer: C
Explanati
on:
• In his independence day speech, the PM spoke about the expansion of
the National Cadet Corps (NCC) in 173 coastal and border districts of
India.
• Under the new initiative, training will be given to around one lakh new
NCC cadets and an attempt will be made that one-third of these new
cadets, will be girl cadets.

National Cadet Corps (NCC)


• The NCC, which was formed in 1948, has its roots to British era uniformed
youth entities like University Corps or University Officer Training Corps. It
is a tri-services organisation, comprising the Army, Navy and Air wings.
• It is the world’s largest voluntary uniformed youth organisation and
currently has a strength of around 14 lakh cadets. It enrolls cadets at high
school and college level and also awards certificates on completion of
various phases.
• Headed by a Director General of three-star military rank, the NCC falls
under the of Ministry of Defence and is led by serving officers from the
Armed forces at various hierarchical positions.
• Each school and college units have Associate NCC Officers and cadets are
also assigned various leadership roles in the form of cadet appointments.
• The NCC cadets receive basic military training at various levels and also
have academic curriculum basics related to Armed forces and their
functioning.
AVAIL OUR HELP ON INSTAGRAM/ TELEGRAM- DEFENCE MANIA
DOWLOAD “DEFENCE MANIA” APP
FOLLOW OUR YOUTUBE CHANNEL
CURRENT AFFAIRS QUESTION BANK

• NCC cadets have played an important role in relief efforts during various
emergency situations. During the ongoing pandemic, over 60,000 NCC
cadets have been deployed for voluntary relief work in coordination with
district and state authorities across the country.
Significance of expansion in border and coastal area
• While the NCC does have units in several border and coastal districts,
officials believe there is a lot of scope for expansion.
• Restructuring of the NCC was one of the key recommendations of the
Committee of Experts (CoE) headed by Lt Gen DB Shekatkar (Retd),which
was constituted in mid-2016.

• In the coastal regions, where youth are already familiar with the sea, the
training will increase interest in careers in Navy, Coast Guard and also
Merchant shipping avenues.
• In the border area, the trained cadets can play an important role in
supporting the Armed forces.
Way ahead for NCC
• There is a need to modernise the NCC, to make it more relevant to the
new and changed times. Further, there are plans to make the NCC
activities compatible with the semester system of the colleges and
universities.
• NCC has a dual funding model where both the centre and states or union
territories provide budgetary support. However, many states have shown
lack of seriousness towards this issue.
• Hence, the CoE has recommended that along with Defence Ministry, the
Home Ministry and Education Ministry should also contribute towards the
NCC.

66. Consider the following statements about University Grants Commission

AVAIL OUR HELP ON INSTAGRAM/ TELEGRAM- DEFENCE MANIA


DOWLOAD “DEFENCE MANIA” APP
FOLLOW OUR YOUTUBE CHANNEL
CURRENT AFFAIRS QUESTION BANK

(UGC):
i. UGC is a statutory body.
ii. UGC disburses funds to Universities.
Which of the statements given above is/are correct?
a) 1 only
b) 2 only
c) Both 1 and 2
d) Neither 1 nor 2

67. Consider the following forces of India:


i. Assam Rifles
ii. Border Security Force
iii. Indian Air Force
iv. Indian Coast Guard
Which of the above forces do not come under the jurisdiction of Armed Forces
Tribunal (AFT)?
a) 4 Only
b) 1 and 4 Only
c) 2, 3 and 4 Only
d) 1, 2 and 4 Only

Answer:
D
Explanati
on

AVAIL OUR HELP ON INSTAGRAM/ TELEGRAM- DEFENCE MANIA


DOWLOAD “DEFENCE MANIA” APP
FOLLOW OUR YOUTUBE CHANNEL

You might also like